Sunteți pe pagina 1din 86

Universidad de Piura

Facultad de Ingeniería

curso: gcoNóMÍa (Eco)


Segunda Práctica Calificada
Fecha: Viernes 18 d.e setiembre de ZOLi
A¡relridos y Nombres:. ....t!.qPvw" Sagde {.".1W. fualrrE
La presente práctica tiene una duración de t hora y 45 minutos.
EI puntaje'de cada pregrrnta se encuentra especificado en ca¿{¡ una
de e11as.
No puede tener apuntes de cIase, ni otros materiar-es
sobre er tema . §E AqEPTA.fórmulas, EL uso DE cAtcur,ADoRA.

I. RESPOIIDA, CONSIGI.IE O DE SER EL CASO SU RESPUESTA AMTE


I.AS SIGUTENTES PREGUNTAS :

1 " fndique a cuáles de las siguient.es funciones nos estamos


refiriendo más abajo. consigne ra retra que corresponde
dentro del parén[esis: _---- (1 p)
a) función de Cosre Variable Medio.
b) Función de Coste Total Medio.
c) Funcién de Coste Total.
d) Funclón de Coste Marginal.
e) Recta Isocoste "

A) Reraciona la cantldad de producto con ef minimo coste de


produccién" (r,_.)
B) Disminuye en tanto es superior ar cMn y es creciente
cuando es inferior al CMn. (& )
c) Muestra er ritmo al que varian los costes de producción.
o9 « t*l
D) contlene fas combinaci-ones de factores que 1a empresa
puede adquirir, dado er coste y ros precios de los
facrores --(
E) su punto minimo que es inter.ceptado con ra curvaJ-; de
costo Marginal se denomina óptimo ce explotación
it--i
2 " EL Benef icio Económico es ..nLtiln.f,.gíá ef
porque icio Contabl_e
I,vryuE los
ru5 eCostos Económicos SOn
ob LOS LCOnOmLCOS _ ff.yt.c,.-..,¡i¡ ñrr- los
son
....¡....¡.".!¡...-i.r..... v*c Costos
contabf es. Estas diferencias suceden' po.q.i. los costos
H v^n^ñ: ^ll --l
^^^

(1 P)
3. Consigne si cada afirmación descrita a continuación, es
1
\
r^ r ^ ^ \
\¡¡-,+)á 1r-
ver()dL)tiLd -
o rdtbd v d-demáS Sustente La r:azón de Su
respuesta. Se precisa que s¡ilo q€ etorgará puntaje si cada
respuesta se encuentra correctamente sustentada. (5 p)
a) Quien pague los costos de ün impuesto dependerá
principalmente de la voluntad de los legisLadores, Ya
que ellos deciden gravar a los oferentes o a }os
demandantes del bien.
b) Los subsidios a productos de primera necesidad son la
mejor forma de ayudar a los pobres de un pais.
c) Se sabe eue, el establecimiento de un impuesto genera
una pérdida irrecuperable de eficiencia (o pérdida
social) " El1o se debe a que quien recauda el impuesto
(e1 Estado) hace un uso menos eficiente de 1os
recursos que el que podrian efectuar las propias
yu! ^-as.
rvrl(
d) Un subsidio si.empre es meior que un impuesto, dado que
esre nunca provoca perdidas de eficiencia ya que
^^!^

mejora el bienestar de todcs.


e) La pérdida de eficiencia causaCa al apl icar Lln
impuest-o se da poroue Ios consumidores tienen que
pagar más.
f ) Cuancio se ciesp-a:a la c:e r--a s ) : :a::t -a =- e^lel¿:.--e
del consumidor y no eI deL prod'-tctor, dado que este
último está determinado por la curva cie demanda.
g) Un desplazamiento en Ia curva de demanda no puede
provocar pérdidas irrecirperabl-es de eficiencia.
h) Et gobierno peruano rro subsidia productos de prirnera
necesidad para Ios más pobres "

4. En economÍa la ciistinciÓn entre Corto y Largo Pfazo se


establece únicamente atenoiendo a Ia exiscencia c no de
. .1i:.\tltx.*:*.....,-.i;*p. Por sr parte, se dice que
existen Econolmi'ás de Escala Crecien¿es si al variar La
cantidad utilizada de t s los factores productivos en una
determinada ...4.1-3..F.H"1^4.ru...".., Ie ,cantidad obtenida de
v producto varia '¡ en' una +rl!i;CI(.L[e;r¡."..,ff¡.1r..\+A-i...". En cambio,
existirá Economias de Escatfa'DecrecientesJ si aL varíar' 1a
cantidad utitizada de.to$-os los factores productivos en una
determinada ..'§IO.p.0.(WW.....*, 1a c-antidad obtenida de
producto varia ér-, ,-,r-,u"."....-¿)Iog:0-{s¿nryr*:fl!\U*.A.L...... (1.5 p)
tr'/
5. En las lineas punteadas consigne las características de un
mercado en competencia perfecta: , a)
..........?.od*x*n§.....§rr,MI0E6-..., b) -. ni/u*chÁ"*; .co.rü¡.r,t¿c,ruuCs-e{c....,
,9 c)y e) ...jcfúl.,elc.g) .\h*^n"d.L aLa'1.*M...-..", d) ...nn¿*c¡¿os.és+Ut.*.\p..s......
..nlr*dÁns...J:.t(,r+.q.s...."co-mfl*r:ne.,A*art»t...." -(1"5 p)
rr. DESARROLLE Y/O GR;AFrQIE, TNTERPRETE DE SER EL CASO, Y/O
CONSIGNE SU RESPUESTA A¡{TE I,AS SIGUTENTES SITUACIONES:

6. E1 Congreso de la República deI Perú conoce, por intermedio


del Ministerio de Economia y Finanzas - MEF, que Ia demanda
y ra oferta der mercado de pescado fresco está determinada
por las siguientes funciones: p = 8,000 - 25e¿ y p = 500 +
// 5Q"
r/ a) Calcule y determine Ia cantidad y el precio de
equiribrio de mercado y e1 excedente del consumidor,
productor y total generados en equilibrio. Grafique.
(1 p)
b) Suponga que el congreso clecide gravar este bien con un
impuesto unitario t 2, calcule y determine ra nueva
cantidad transada en este mercado, además del precio
que pagará el consumidor (pc) y el precio que recibirá
el productor (Pp). (1 p)
c) Calcule y determine 1a recaudación de impuestos que
logrará eI gobierno, Ia pérdida de eficiencia
generada, quiénes soportarán Ia mayor incidencia
tributaria y consigne la razón de esto úttimo. (1 p)
d) En un solo gráf ico ilustre 1o obt enido en l-os
numerales b) y c) " (1 p)
El Ministerio de vivienda, Construcción y saneamiento del
Perú a cargo de Mitton Martín von Hesse, ha determinado que
el mercado de la vivienda en perú se encuentra explicado
por las siguientes curvas de demanda y oferta: p = 2OO - 2ed
y P=2Q+ 4Q"
a) Calcule
-
y deternLine la cantidaC y el precio de
aarli i]-rri
crj*---LI-LU a u€ rar¡¡,1
Il.=f Ua(jO ,' .,^^--l^^-
^^
^ _y E-
^l cn-=;s._-€ Ce_ CanSU::-r.i dOf^,
productor y total generados en equilibrio. (1 p)
b) Suponga que el gobierno cori el fin de estimular la
vivienda propia en este mercado decide hacer un
subsidio a la construcción viviendas de s : 15. En
este nuevo escenario, ¿cuánto será et precio que
reciba el productor (Pp), el precio que pague el
consumidor (Pc) y l-a nueva cantidad de viviendas que se
transen en este mercado? (1 p)
c) calcule y determine ta pérdida írrecuperabr-e de
<-- eficiencia, quiénes reciben un mayor beneficio
(cuánto) de los subsidios y consigne la razón d.e esto
último. (1 p)
d) En un solq grafico ilustre los numerales b) y c). (1 p)
8. Se tiene la siguiente función de produccién a = !gK2L2
o. o5K3l,3 en donde K : 25 unidad.es de capital y L : horas de
Trabaj o por persona:
a) Calcule y determine el número de horas por traba;ador
que deberia contratar la empresa (t) y el nivef de
produccién (O) si la empresa encuentra su punto
óptimo. (1 p)
b) Calqufe y deterniins la máxima producción (O) que
podría lograr 1a empresa y e1 número cle horas de
trabajo por perso.ra (L) para este niver cle producción.
(1 p)
UN¡VE HSIDAtrI trIE PIUFIA
I
[Corcur n
AST.NAT,RA TRABAJ. No L
Pf: S-ott' NoMBRE r¡a[¡r,^n tLl"q"r',, ]f"qqJq FECHA /f ¡. n,a
?0'. +-r__
Vsrr 4i
FIRMA DEL PROFESOR O AYUDANTE CALIFICACION RECTIFICACION FIRMA DEL PROFESOR

, /trl
\\/ / I

Vi
/\,

/'
)-l*Fda,r .ro *.,ip#*.'fffi\:lilL
'
flHM.s.-,il%darcexíru puo:E dqiv,,ida:
s4g y:ü;;tilkd' '----
ówu.ú\u. {*uo,ro'^ bte"
{-i]ffil ;;* da^d:¡ o-
__

t,E:Mrm*x**h,ff*k KTq.r Hur


J Ya,\so r Va-qle o\
C) o{ remrdq( , h( irvtpurths LOs u}9 b"(ftne§(
.¿<\odo
¿* ú'aÑ.*010\ de lcs ne.etdr.des de '\t¡s porfoms' '-

d) r.r**ff subsdro .}av,,rbrvh pror{,olo,«s ds' si*Y-


p) É*U" . {e^wrbre,rñ cornbttdo[ €Fceclen@l produ<lo(, 1.------__:

n¡ perrrarno .A.t S*bSf&taf\rdldof de flunrtetQ


E n,
qobter
'-
LFTL'"'
no.".tdfr Pl'\ot Putes'
- rtlt

r) r) e: Is18 Lt- orof k=tj b) 0= r?s oo L- l\?) rtsí LP


*le=r t qz:f ¡'- o,¡o§Czsf L3
(Q= [ lrso L- -at tles t-?
t o¡b

[r= aZsoo L {t1\)tX t1


.-\_.
(Ú o. I ooo - u:Qd, f = Soo {Qo

-,\ ?= tos- Qa ., ? ,-^ foo-l e9


/rs \
5

lQo= 3a¡ ^ o,O'tP-


I
§o' o,af-.ll
/'
?f-o- olOQtteSC
Q.d =
¡e§ -olPq 3= o:a?*toO
{§o ; o, }q?
Qq.= t5o ¿2
,/ ,r'/-'
Ilso =?ert_/ ' üruvroto Q'L'o '
'

*-;t'
Cuavrot¡ Qo=o \\\ o= 3a,q - Ot .{ P
' '¿.0rlt:\0§' o=o,Lt-too o,dP 3to
_-

\o § = 0',aP P-- rooo


/'
5oo --?

g " ju»-oP\f
§.
tlo I P = (J a so - s or)t*Ql' lse L\o ,

\
- o,ott /rlto \ - o,
\as",/=

io -F,.í=ygr# 1/
lu

t.
q)a
I

:[ó),bf
r,u-
\ a,tf-

ó r)so t,P
t I

\?'l1ro

A
luq,3j¿ )so \

[]a P= §¡*QQo
Qo = e - 5
f=Eoo-tQoL
Q.a= too -o)<l
,[§o-oiS?=O)2SP-S
-¿oo=q16P
190 = ft-,
Qq= [oo*o,1trüo)
%-@,,
L1-'
Qo = ores ?- S
o*\Qo =o ,\3,aJio
t
.o = e ¡1:?-5
...PC tt o 1z
f . c =[§n"-f .to) (eo) 5= olz<§
cL
^ *, Qp
f , c= Qoo, t--t' :)Xuo\Aoo
- Y,l = A.)oo ,

<\- - o>S fg9\, - a) 33


to= qps (f" J, I) \30 /
i') )=
Ap= -li {} - ( lS)
^-5i 0)
| (rs- 5,o\\>Nl,997-
flp = J r{CI
o .,1)) t ?r13 \
?c-- lqo-6,o[ : ,t3L't¡\\u
ru91QQ= t-0 \qqÜ
itlutrs = Qd z, Gn< urn^rd¡.o¡'"
- ------.-'-'-

c\
rl
(Jqq,Qor- l3qrE 1 ) ( o,qqlr J .5,ot - q33q xloo-3379/
= r6y(1s " i5
Ldofft bewg¿^ L»:
''lPtoo¿^c+ote¡
Reu"l.rq^ .^ ' 1 qa "

{rod,r;-cftoy.sc , Ya
o+u'r' t*. .-qlc={. ';d'Ñ-
"'- ts *V
=-D)&G*¡,,=ae,'d.
o(o opcrta .{A (vteMo( {}"*ico.=:Y
d)r
efft
\rq1)11
\I 1 tec
1't"
1'\u,qi
,s)0\
.) '¿
-É.. -
.A
UNIVERSIDAD DE PruRA
FACULTAD DE INGENIERIA
ECONOMÍa leCO¡
PNÁCUCA CALIFiCADA N'2
PiURA, \TERNES 2 DE SETIEMBRE DE 2011 3:00 - 5:00 p.m.
NOMBRE FroüL

1. Considere un gráfico con Ia variable precio del bien X (Px) en e1 eje vertical y 1a
variable cantidad del bien X (X) en e1 eje horizonta-l. Grafique una curva de
demanda y explique crrá1 es 1a diferencia entre cambio en la cantidad
demandada y carnbio en 1a demanda. (5p)

"muest¡e
2. Utilizar-rcio el Gráfico adjunto los efectos d.e la intervención del Estado
en los siguientes casos:

a. Imposición de un Precio mrnimo. (5p)


b. Ei Estado actúa como comprador. (5p)
c. En un mercado que existe i:nportación del bien X, se aplica un arancel.
(sp)

En todos 1os casos muestre 1os excedentes del consurnidor y productor iniciales,
posteriores y los efectos netos de las medidas.

Total: 20 puntos
¡-¡NIIVEHSItrIA[f
/-, l- i

ASTGNATURA fr:crrron^i6" TRABAJo No ad- I

\\
NoMBHE r-q'\p\c, fua Rmc\Ví^ - FECHA eA -Og-2Dll
I'
l

,-o -') r--'.


-/ \----'
? FIRMA DEL PROFESOR O AYUDANTE CALIFICACION RECTIFICACION FIRMA DEL PROFESOR

lon'
-.'...,.--...-'--.".-,.-_----..\

\ iY, o &,"..r{o- J

\[-l \
) \)l\\\l\ \ \
1
\\ {,_,\
\,"i -\
\ñ I
il\a
Nxolt----\
|
\ A
'\
\
\ '\\, il_-- - -- r
\ t'\
ll\
\
* "U"^*J,"..
*8^do':\yÁuwo¡q\,nen[
I
./
I \-l
l-\ \#
Y-> u^ hen Y ,\
\ 0* \ rtr-h$)sQry, U\c,Yn9q616alic.
cr[ \
.-\., 1

F-'
'ffi Y"J"t''Lu
I ¡ I .,\ z
y *:,,^{i
M.\_/s.cLo,n_n¡!*ro/*
lr*i
* "*1,\o'o-'(wch''-n'l*'a'L
l¡ lr,\ l;r'ont'"-
, I
a
,,.----^-=1-- §t&dr;\
,., *^\,
)yo=-,t=1?*,
\, * !-, .. ru!ñ(¡dr& d"*^dovh.) j:lon ,1'
. __-q^^\- d¡rer^» o,r-*,r{*j
L_ " .,1"1
"X a¡s t dur.,^r"C. aLr.\t,ru.^
.-*-,
/.\\.
G*-a-p ['\r\,r,.,.^s, ¿o t*^\rru *\o &,"rc,,.¡* , 5r",('.* r *V*[d. J" A (Lxue. á" 11,'-------_._
-------l---..r
(w'u^^tl"t
,trr+¡¡th e..t
.e)
.1

--/'l er*uuryr d\¡ u\tnr-


¡ri'¡ur¡ p,¡\o
Ynrdu:or( 1$ E.\-r,us!+ o- \o i37.,srdta. U\-o(.olix 9*., i._r"&
-{ r._-'
* -t d[ Jrn.,*,dr*L*^ ¡ ,"t ,r-*."f,.' ln. *-*r* §.". áJ_o ,,,' * *i,.,
-:-' - "
; cl[b*^vuu*, ^^i.^uw
to, raqu.rtrc\0.. ...:-.--.--
^
[]uv\, ,t, ,,*,\.-.* a^J";I¡. j & &,^-,r.{. fir= *[*.,,r-* *.y]- ^*..^r"*,f,o 0,,¡t .ll,
n*L--ri o '*..['ru- \+'L\a
^^u.
cr cu-",^\.. ..,( '' *\...a fnrib;'; v tLc^r' yt í* Aij:¿:i,:;
i^.ril, rn/-lú()'r1 .....A:r._i\+', -!É1.,
*-_._-

¿- I l
' ,-' \-'
\
')-\
a4 iXo
D*-**-^ cr..[.J*Jü*"d. .Li Xo.
/i;ood"ñ d"\
"*e^*a* *dd"
I
'" ABk" f-----=::

h k"Q*'o
1

1
&"^""üo * p*Oe2 .,*^d,ríL c^
sn .[

-/¡ A

ü.ffiY,^$*S{
A
trvl¡rr*.to" O l:(ro,c.{
d-'-^***'u\ .i*q x p.$,* J tr
- .v- c-o*r.renk §*\LL'(tLt4
u a"G
,pr<c^s
,
t- \ u" y ."
§§-_colrr;
*tre.uq M
rnrlrc"oe_.-;S^ú..^ PT"' !" tt" ?*Cr\ .rryr{_3
o..- n ,
vt\-isL¡dfir1€
All I lr . ñ
ü1{ plOd.rO}W
Lejr
P|-\-cL',,
Feo.§ 1 4b
IFoots
"^ \ó arñ z
\
\

"GJ-

ñl D
.'É.
i
?xo \

i
l"

d, d) )
X;-, :2
t_
I f-teC n'poiw
\'in-l-vryv-
)\ _*ts

! ,rno-t-,* ¿r[n .t[o=*noí.o-1ft a^on^.[r*. \]n .pso^é rr\a{oi -\ p*aé & *¡,-,ú6.,,* r.ry) t}-..,¿,,t,

^
' Clndo.n\- ll \ r' \n t-.,na
'n..o'!'¿ cL'f.u'or
Cu.a '-
a
Crcn r0O ¡AC ¿ro y q l¿,rna L'¡no 'nuQ,!.¿ I

[^^rr.¡, .[ (*r"U-) Yhú t ár'*n.a, o.i' ,,.;af


"*d .

**W-J \*"UrU vr<^ Ar. ,*.i ol§oc 'kd.


nt)/'\
1"
¿/",.,^*r,u 'u-!- y,t,ro(,

\e \e cLr,,:,".t&P
.T.
q-lo.ntor: /^+U,\f
,
t\ ü^
irb",.!(-u:i* =J.g\.* JAgK
-F ó c' -- r\r) l.t*U'^", luru¿o J",{- p.rc.cie II
-
- II /
t:t¡Stt: .t I

¡t I lPn*"u'\
\le^s.-* .J 1:.., ¿,',t"[.cn s Xl
'p'=, Pxl
.ü -t ! t,i -
ffu^X -o.lx nvQuúa^ 9,uutco ¿-rrt
irH,'lÉc' r \LE'- -x-:*or\\z = :l

i^ t. l'; ,rnü-,,,,
I /
-
!
'& /,* -./ oá"
\.,-, * ^Al:..:-{¡ (d., .rl o}O,J*
r
I rrír-L-t\t"tü .¿o of
&i;¡rW
"t]
*^\",.
f\r!l ) ;r: !v\*. \t{ cl!_.rf\(rnd.o ül'rcJlur G '*
$* \ \r""
-X') \*^i, (§n^,. .! ú,^rr^ 1-) -
-\ J Br?' \ / \
^ at 'rn lF¡q \+e.ñ Sc tfJ.
7\
| .. ,.u^.. ----f l--^
'-Yl t'^ L.'LU(unt'c.lrlc c,t '
\
q., J .[, ^rru^dl)
tr ^r 6
'\)o^^r¡^dr']
-t'l '{r'\t:Dr

DC: N$
A t- Px'
Lp= P,*B P"'D,/
\ tc -^ t,A

p.[xi--d)

§,"*.\#tu *=-e.\q Ú".9" ,r, X, ,, n%T *'"Jiln,


^
f dr"",*J.oJor [\(r) Y"
-/
I I ---"---i---.-\
_[-\.
l[\ \.rrr* d"
'"yÑ.*S" to.n o<¿"nuj '" ',
0.u-,.,!. =
A 4r* *k^w * L '"r,.

ftL
x') ./,
'-'
\J '
S
i t\
t-\
\U
G,/ra,.'Ci\ SPt i-'eu- ?o-Dfl =\-
!n,
[./1r"^A *.**'al YuY
4-**;;D
¡ al
tls --+ Qs
lal/
A rrulW ,)' Jr¿h.
I
&r[ o*-.*\
-
t} G t^l
)@d§ (n +r '-'- l
r e +\-r\ ü J-.,r{-,u^ J,^J a"*..*./*,
)*. F eh¡//nv A, l*."1,--' ^,,,

:)

3 fu'-*.

Uo'urot- D ,r^Y^^
'D*"
Á g**^*, Q*"t/.-L ru ,b@.
cvwn,eaj
\/vwt\!!t-- .a gan^r.(,^
¿ ( §o'
'.-.¡ r-_rD -{is'),
-" r ¡ N"rf-Ll._-_l:_- ,
'c('
'tf *,a,! ?* <t.ru, cc,,,ftc/r,,1 ú^: úc/s ,-¡rrJ^or*, É^/*í .*íq -*-F*
'\.
a,too;rp*fl4
lry" "/in--D
¿' -/"*.,," (,J gr"t;,-, P;gr) &ú ofru -2/u^ cJ.a.Laao> rAf,ó,yr"t\
I

L.[NIVEtrBSIESA.E} Mffi Pfi[".¡TqA

ASIGNATURA €IAV,O'¡N(O* TRABAJO ¡o 1',1-

NoMBRE kaul«- óutrios Arnu,''u''. FECHA '13 lc-) 'i'lt't]

a
FIRMA DEL PROFESOB O AYUDANTE CALI FI CACI ON FIECTIFICACION FIRMA DEL PROFESOR

/'-i''
(J, [r)¿ . Jc cCOo - ZoC I -7 ,* / (¡ti\, bri-,
f^) (ic:-roi ¡¡,¿;tt) 1-l OO f
/
,/
4itf) /'
d,..r, /
,A t/
,/
-,.,,u /
\ // /-.-\
'/ /\.l
n.-
[i= '{r>
\{
/\ ,r
t
l/ \
,, ,/-'

_._.__,r \\ 7.
*4oOoü » rl
?'0 o¿:o 'J C.l-,.,:

( lr)

e ¿= .- A¿i i)

o .r= '
^t A ?,ogt{rp
/a

, ('..tr-,i
,,)p \' \f). t 1_-
-\1)- 1
^tt,c,?-,')
' ' ': \- \ tc.-¡.,oc{l
,*.\*-'l1^ -
\
I ._-\ t-l IS*(*-.)
i ''p,.í*-,rul- - : )o\:c
]'l-
l.'
,' ) \1
l- t\
lt,l
'i'., -- 1i"-' r '-\ P- 3<:c li'
I ' y' ' '-'

,no,l '"' ''

(,) \'-"
P, = .r, xt*úl=. l)-i5"L, / i ",cüf ),'\
L'V -/ ii,.
\7án ^,au,1 .-rv n.^^t ¡,-'¡,ziL ,¿, t..¿tut'-.vt,..' ,,' i.,'. '

(i;, .!j,!*-t.,,(-;l_-. ívrv.*r.lLv' ,VLL(4.u,, 1',; ,¡,.t.,:;li,-r,),:.,..,- ¡.i.t, t', ..,i.', ,,:\.t.r:
C,t,. \'1J)
,\ \
I ,'-1 ,n
* \ (' ut:<;
/lr
'l \'lí\\c\
,r,i - I OAAlr flÚc.(Z¿'t.)
|
AYJ§'J&yod,u
Ls,) ()i-) -' N b,.Aac:
\' '" 't' '
w^u-- LttÚu'¡ -
¿L(y'
/, \
: W"::¿') t:9:::) , (l40,oo,s
Éci
r1

g
,l,2ic.,C^:,.,
\fl
{
'bO n [,bCetO '; Ol G o, oD)
Iurgr<nol (\tor,\t,
/'
,¡\
, A [ = bct+ {-?' -v tAJ'wz-.2 (iJc'-t- -- '@l;|:attis' 2-
l/ \-,-
tl"l'r-
't'-)e \/ i[)
v ',i

ti/ Ll
' l,^^ ?
I ¡ ¡7t7t,f
't
1: ¿u¡,ltt-ai

IC ,a+f5t; 15 "il\"rl-,
-,.U) A

t\ t, f. i) L-r D. .--l '1r +i- /


t/ 1. r >-t
"t
(,
F L)

J - t'.,

)
,ir ')-üa,
t,/
?-ui,l:J.
i -r(-
i¡1ú
,z lra; i,)i,* ,
Güte+i=t(
r\ /';" ,-r )
i-. t- /
¿+D !,;
"'. D
leoao 'k:e.t'.. f;r)':.¡,t

ltl
= i[
(''¿coco ,\zg,c:o:,
'f rü
,1a\l¿a""":la'rt W
?o.,-)-(bt)
l«\
\. -/
(Ul(Ur,c. cL 1,u'¡u"Ll.,-, l

(c) t.,\J
f ,{) l1. ;.-i-,
1-)¡ t
" .,. :.- ,. ..!. ":.i
4'
')"1(-"'Y ' /)
U(',,,-.,1. 'L . ,, l\
'1
.\[l VCtLOy*,lt [.,.;,t ;¿;,; L^¿,.,..o., :.k
1 r.-)- \ii lu;lC _i \
t-\
l q,') L. ,,i
d *___),o.,.¡
L 'r5
.r'§/t:
(c) l,,u-n'' ,rl

5
\\...,
b uo^ ,.1 --\
I lr I
l

-r.
4
\, l- -r
¡¿rin7r'q ¡r t I ,, ), L t_út,
jp.t) r-.,¡.l,,., ,Q!
" IL'

3 L1; l- t, -- lt1. .
I
J,' tl
{- f) ll,
L ' tt) ,(/
ttiversidud dc Pittrn
(J

Fu c ult orl d e I n gcn ierío


'r,,2 f
curso: ncoNóuÍa (ECo)
Seg'unda Práctica Calificada
Fecha: Vrernes 1-3 de setiem]¡re de 20f3

Apellidos y Nombres: Ruraos krvt{K.


La presente Práctica tiene una duración de L hora Y 40 minutos.
EI puntaje de cada Preg-unta se encuentra especificado en cada una
de el]-as.
No puede tener apuntes de clase, fórmulas, ni otros materÍales
SObrE EI IEInA. SE ACEPTA EL USO DE CAICULADORA.

I. RESPONDA, CONSIGNE O }ARQUE DE SER EL CASO SU RESPUESTA ANTE


I,AS SIGUIENTES PREGUNTAS :

l-. Cuántos más bienes sustiLutos exr an de un bien, iQué le


ocurrirá a la elasticidad*precrg/¿¿ la demancla?
I ado , ¿pár qué se cLrce qlre - -los consumidores Les i¡teresa
--,-engan
que loi productos que adquieren en el mercado
valores altos cle e-r.astl-ciciad*precio Y,.. si-n embargo, a los
1ñ (2 puntos
tos )
jl.l.'
,.t ,^ '-

..rfr,:t::l!:¡<..',......

2. Por I\ Glr" tic irlad-rert La Ios \tf qG¡ pueden se ' : a)


... lvll.$-lb.; e b) ... tJvt(*-ff4^1-c!.Hc.5.' .,en e1 orlmeí

uif ;;=;,
caso, sr ;i l\ ;ie1 varor clc ia elasr.!iáu.a-rent
valor de eLastrcrál¿.d-renr u" l" ,Á" enc¡entra
enc rer.r ra
.r-,tr" 0 y\ r, ios birenes serán : .'-.,[,n.t.l.t];.t,*t- ) z/-"'." ' "'y si e1
va]_or eS ma yo r / que i ,
....... ./t.l.q,'5.i.i,,*
entonces lo.V baene s selan
L punto)
(
\/.
Po lasticidad-cruzada los bienes pueden sel:: a)
si la relación del Precio de un prodr¡cl- con
_ñ t^
Ld demanda de otro producto es dirccta Y b) !/..i)
-i Ia relación del precro de un Producto con la da de
ü\ (3- punto)
ro producto es inversa.
Marque Ia respuesta f¡qQBP.ICIA.t (1- punto)
Un subsrdio otorgado el gobierno incentiva aL
\--' productor y desPlaza laPor
fil
curva de oferta de dicho bien
a la derecha.
\\il si la relación: tslasticidad c1e 1a demanda/ nLasticidad
\. )\ de la oferta, es pequeña; entonces eL subsidio o
subvención beneficra mayormente a los pr-oductores.
c) La proporción de Lln subsidro que terminan recrbrendo
los consumidores o productores, dependerá de las
7

elastrcrCades l:elativas clr: ia oferta y -l a demancia


d) La carc{a de Lin rinpues to recae más en 1a parte de1,,
mercaCo (of erta o deinanda ) que tenga la curva menos
eIástica.
,/. Ia respuest_a CCRRECTA:
5. lvlaf'que (1 punto)
ál Por La Ley de I a Ut ilidad Mar:qina ,l Decrecienre
\t rr entendemos que cuando consunimos más de un bien/ menos
l'\!, satisfacción adicionaL reporra cada unidaci de e se
bien.
b) La Restricc:-ón Presupuestaria se ve restringicla
únicamente por los precios de los brenes.
c) Uno de los supuestos de 1a Restr:icción presupuesl-aria
indica que la r:enta se gasta parcralmente en bienes y
servicios de consumo.
d) El cociente de los precl-os relativos es drferente a lo
largo de una ilecta de Pr:esupuesto.
6. En economia a distinción entre Corto y Largo Pl-a zo se
I
e s table ce mente ar-enCienrlo a 1a ex-rsLencia o no de
ta(tonq ¡ijc 5 fl,u Por su parte, Se dice que
existen' Ecohomias de Escafa Crecientes si aI variar la
\, cantidad utrllizada de .r-gdos los factores productivos en Ltna
determina dat ...P.t,,.'í-)..Q=:--........., .l a canrrdaci obuenj-da de
procluc to varial
va r ial en *+9*laL(..... En
¡na--.=TPh:fdd¡* *+4Nav.-.....
en ¡naffie..4?l* En cambi o,
o,
^-,1.,r; -< Economias de sr"lulto
gAAJLI!A 'D.;;;;i;;;J;;i or varf ar I a
cantidacl utilizada de todos los factores productivos en una
:....,
-- Ia cantidad obEenrda de
producto varia en'una f. :.....*itr (2 puntos)
rI. DESARROLLE Y/O GRAFTQIIE, TNTERPRETE DE SER EL CASO, Y/O
CONSIGNE SU RESPUESTA ANTE LAS SIGUTENTES SITUACIONES:

7. Suponga que la demancia y oferta de equipos de scnido en


Piura se representa por medio de las func.Lones:
Qd = 80,000 200P y Qo = -10,000 + l-00P. Donde P es el-
precio establecido en dóIares amer:icanos y Q la cantioad cle
equipos de sonido en unidacies al año:
a) Determine y qrafique el precio y Ia cantidad de
pquilibrio de mercado, eI valor del Excedente del
Consumidor, deI Excedente cleL Productor y del
Excedente Total. (2 puntos)
\ 1-) b) Suponga que el gobierno establece un impuesto de US $
60 por equipo de sonido. ¿Cuánto pagarán ahora los
\,- consumidores? ¿Cuánto recibirán ahora los productores?
incluya estos datos en el gráfico de a) (2 puntos).
j
c) ¿Cuánto es Ia ncidenci a del impuesto que asumen los_
consumidores y los productores? ¿Quién asume la mayor
parte del inLpuesto y expliqué brevemente por qué?
(1- punto)
.1-,rán'f-a a(- al
d) ¿vualrLv valOr de_L nUeVO ExCedente del COnSUmidOr
y el del nuevo Excedente del Productor? ¿Cuánto serán
Ios ingresos fiscales? (1- punto)
e) ¿.A cuánto asciencie el valor del nuevo Excedente Total
de la sociedad? ¿A cuánto asciende Ia pérdida de1
bienestar o el coste de eficiencia? Muestre esto
último en el grafico de a) . (1 p""i")
muestra cuaLI-o comi¡inac:-ones de Ios
I
T-r
!cr qia"io'rl-e
urYua Labla
bienesXeYParaun determinado consumidor represent'rl-j-vo :

Combiilacrón Bienes l-clIIL Cant l-dIl


^-*! u Cant Cant Cant
3 4 5 6 1
X ?
A
Y L2 6 4.5 ,{:
-1 .J 3 2.1
6 1 B
x J 4 5
B
Y )-2 B 6.3 tr
.) 4.4 4
o
X 5 5.5 6 1 O 9
C
Y L2 9 7.8 6.1 6 5.4
X 1 B 9 10 11 72
D
Y T2 8.5 1 6.3 5"1 5.3

a) Grafique }as curvas de incriferencra de Las


combinaciones A, B, C Y D en un mismo sistema de ejes
y consigne correctamente los valores ' (2 puntos)
b) Determine la RMS entre todos l-os punt-os corlsec'itivos
de las c¡-latro curvas de rndiferencia (Use (1- tal¡1a de
ayucla abaio mostrada) Punto)
c) Suponiendo que P* s/. 2, P-' : s/' 1 y la Renta 1*)
s/. 76 al aia (todo lo qasta tocto en la adquis¡-crón de
los dos bienes); trace en e1 gráfico de a) la Recta Ce
Presupuesto y consigne los val-ores en ios extremos '
Asimismo, d'etermine y consigne las cantídades de
bienes para las cuales eL consumiclor maximr za su
utilidacl o satrsfaqción total (óptimo) ' (l- punto)
d) Sj eI P* disminuye cle S/' 2 a S/' L, permanec:-endo
constante eI Py, los gustos y la Renta (R) clel
consumidor; trace en eI gráfico de a) Ia nueva RP y
consig'ne los valores en Los extremos ' Asimismo-'
determine y consigne 1as cantidades para Ia nueva
situación "" la que eI consumicior maximiza su utrl-idad
o satisf acción total. (1" Punto)

Se sug:iere que para clesarrollar b) utilíce la tabla de


ayud.a que se encuentra líneas abajo:

Combinación Combinac. B Combinac. C Combínac. D

A
Y RMS x Y ,,RMS x Y RMS x .r. RMS
2 L2 3 L2/ 5 ,.L2 '1 /12
3 6 L (ü 4 ,la zf. s.F' 9 ¡b ,/n 8.5 \ j -<
' ,')
.lt'
4 4.5 \ 1s 5 6.3 -V\ /L
lL.-) ./ t) 7.8 7.4- I "l ,j\ <

JE 3.5 1- \/6 5 { 1..j 7 6.'l 1.1 10 6.3 o.:\


6 3 ),_) 7 4.4 ü. t/ B 6 ) .:I t_ 1_ 5.7 o.d
7 2.7 b,b B 4 0.4 9 5.4 lrL-L' L2 5.3 i r, . .ti-
,j,,,=é¡ .;..''-a?n+j.iÉ=Ésé*=,+=¡:-ái¡,:ré*,*iÉi*I++s€.?á"i#+#Fi--4#_,+*.:'3*¿#,¡*

UNIVEFlSItrIA§t t]E PILJtrIA

ASTGNATUR a lF{,Ü TRABAJO


- No

NOMBRE Ftr.lr^ txi Ccl t4b

FIRMA DEL PROFESOR O AYUDANTE


Yrt
CALIFICACION RECTIFICACION FIRMA DEL PFIOFESOR

D,"t
!ÁvL- . {r) r)tO - }üc}
uv P . ü,, urr r.* $
C! -, aw:kz,t!¡-'t-u\ flt e-'4"-ot-"¡"-: ll-r 'i¡':-i'¿l*a'
Qo , 4ü Coc --i {ül {
1:¿

= tQo'
Qc'l"
---
iü r];,; * 2AC t : *
lC ¡:cil 'j '1llr, I ,-..1 D

-v
n.
.- :-,1 -- ^
---.*.'

l1_ -iior-r
_- a.,/

Í-¡kr¡,Q¿¡v"Ar q{.
,4-\
// \ :,'-)I
¡,;ii = '60«lC
,
'/ i lr,1
ii t
r, Loü}]
(-^i
A Olo r»D
-- j

\-L 1]0C
i\\ { {1Do) l

lt \ ,-1
F<--

Go- -{oot:D l,/


, \

! = ¡oO
r.:i,
I
: ai oal (tccJ
e¿.

Lr,¡.lL.r¿i
I
,L i, ;,:, (..= A& ,
¡')
p t,5
AP ,3, 20 OlD
,
e t A

\,O-.(nd ü',a L4 Á^
T* x 'sro
Au1- -
A? ñ. 2-O co)
, Iuo¡ /\
^,)
¿, f - (-3) l^\
IVUr 3OC_+'r9'], '> (,'.r' ("/tt'L)r tñn
I

'
\+Y¡ ,&-rJ
\ \) ,
lt
_-L..
.[(-
[,u ürrn' Lr,s'

4:']i+r¡""
,-{¡J -Jr"''.
¡.t;;
}*-*'
-i // frn'l¿'cL'
'?o *}?*rt; -2'o q0
- ?ü = ?oo V Yu- Pv = lao ÍLaa '

t\ ,.\
==LN1oafu
I II'\/- \\
ty
l-'/ \\ 'r
\
- ql\l i\\,
ri\ , r \
\ --\\
,1 \
,l .tI\ \

\
\'.gp) 1l

ñ.
)
1

l)qIroi

l^\ htJ\( AN.


g- 4b
ü= ü[§* + lltr.)\l \'
)
/\ \lq n
tlo-- ley t .r- ) l^
wt
/ I \
\ \/a
\/ ! lv
\ { \i
, \\i
\ ..V
N {I.J
|r,/v

-,i
\¡ fl ()
: ¡/ --.\/
ti
, ,1 \/
VI \
fi
\,\:
'i
L
i \ -.,
tr-J
t'\ \

li)-r fr' '1 "


* (i *r\ \
ni,r= ! Gx, \ o'\ l'
\ (A .( ,.',
t v, ¿nt
Universidad de Piuro-
Fercultüd de Ingeniería l\,\ "z Y'
1

Curso: ECONóMÍA (ECo)


Segiunda Práctica Calificada
Fecha: vaeunes J_5 de setiembre de 2OL3

Apelridos y Nombres : . !+t:fUlUo,.


J""
0AIfu^Á 8ur*+
La presente práctíea tiene una d.uración d.e t hora y 40 minutos.
El puntaje de cada preg-unta se encuentra especificado en cada una
de el].as "
No puede tener apuntes de c1ase, fórmulas, ni otros materiales
sobre e]- tema. SE ACEPTA EL USO DE CAICULADORA.

I. RESPONDA/ CONSIGIIE O ¡,Í¡IRQUE/ DE SER EL CASO, SU P,ESPUESTA A]{IE


LAS STGUIENTES PREGUNTAS :
/
L. Cuántos más bienes sustitutos existan de un bien , ¿qu/ Le
. ocu rr., rá a Ia eLas iiciCad-precio de Ia derlanda? , /
....".l,1i¿Lr..,Ln/.* ..Li. .t¿,l,.tiL-,.**.&...ti,....i:.,.f
' ..:.r.:.{.,.i.v.,.\fi-r...?(.'..X:.1.t..*.Í!.Í.:!
:'"""-""""J"""""""Ú"""" ""' lrlú cr ro
..i4..!.i:tt
i ¡rrn
rugv, 'n^r
.yu- -'- '' :'^'
qué s3 ;'dice ^':' qiei a lcs consumrdores les interesa
que los productos que adguieren en el mercado tengan
valores altos de elasticidacl-precio y, sln embargo, d^ I(JS I ^^

productores ies interesa que tengan vaiores ba 19.é?


s) _.t.;,
'
- ) /i- .¿-

..C*v1.*LtÍtf-'rt-{A
" .", ,. rl 4 Á: lrt ,Lr, z¡ ,!
OtJwtr{/fi-r|r\ |'KtY A¿^' ii' !;';;:',:¡.';" i¡ "[2,r.+..
r a?
,{ry\ t't aL,.r
ñ2 Por: Ia e-las t¿ cr C-renta Ios bienes pp?den ser a)
,\, .... e b) "..".....tt*i.:...i..1..it;!.4/.t.t.......(... En eI ramer
\\ casc, si la elasticldad;rr-L\la " se q/cuenLra
\ entre 0 y
\ óa entonces Ios j}énes se rán
^r L , b
(1 punto)
3" Por la ticidad-cruzada los bienes oueden ser: a)

s\ si Ia relación del precio de un od


la demanCa de otro producto es directa y b)
si la relación del precio de un producto con I
otro producto es inversa.
con

(1 punto)
4. Marque la respuesta INCORRECTA: (1 punto)
,a) subsidio otorqudo po. eI goblerno incentiva al-
un
/
I
^- oductor y desplaza la curva de oferta de dicho bien
la Ia derecha.
\{ V{:' . \
1a relación: Elasticidad de 1a demanda/ Elasticidad
^Ó la oferta,
LVU €S pequeña; entonces el subsidio o
bvención beneficia mayormente a los productores.
c) La proporción de un subsrdio que terminan recibiendo
lo s consumidores o productores, dependerá de las
e-Lasi-rcirlacles r:elatir¡as c]e ra ofe::ta .! la riemai'ida.
d) La car:ga de un impuestc iecae más en .a par:Le del
rnercado (of erta o demanci a ) que tenqa la curva mencs
el ástica.
5 Ia respuesta CORRECTA:
l,Iar:.que (1- punto) "{
Por la Ley de La Utiliciacj Mar:ginal Decreclente
entendemos que cuando consumimos m.ás de un bien/ menos
satisfacción adicional reporta cada unrdad Ce ese
bien.
b) La Restricción Presupuestaria se ve restrrngida
únicamente por los precios de los bienes.
d) uno de los supuestos de ia Restricción presupuestaria
incirca que Ia renra se gasra parcialmenre en bienes y
servicios de consumo.
d) E1 cociente ce Los precros rel a-uivos es diferente a 1o
1ar:go de una Recta de presupuest o .

trn economia 1a drst-jnción entre Ccrto y i,argo pfazo se


establece únrcamente atenciendo a la ex-i stencia o no cre
.". ..."......r---*:--...................... - por su parte, se dice que
existen Economias de Escafa Crecientes si al vai:ia:: ia
cantidad utiii zacla cle t-odos los f actores nr(1.ir¡r-i i ¡rr-rq rr
cieterminada ,.,"" r'.*:";"Jt¿";i"" ":;
(\.,1i procuctc ¡¡aría en una ":"i".?,"ta"""
L-¡
Lrl ^--r^l
!atirJ!u, ^
\li
\-/ eixistirá Economias ce Escafa Decrecienres si a1 variar ra
I
I
cantidad utilizada de todos los factores productivos en una
producto varía en una .................,:'...... .....::.. (2 puntos)
II. DESARROILE Y/O GRAFIQITE, TNTERPRETE DE SER EL CASO, Y/O
CONSIGNE SU RESPUESTA ANTE I,AS SIGUIENTES SITUACIONES:

7. Suponga que la demanda y oferta de equipos de sonido en


ñ..i
ur d" se representa
r-r- ,,.. por medio de las f uncrones :
Qd = 80,000 200p y eo = -j-0,000 + 100p. Donde p es el
precio establecido en dórares americanos y e ta cantidacl de
equipos de sonido en unidades al año:
a) Determine y grafique er precio y la cantidad de
equilibrio ce mercado, el valor del Excedente del

q( Consumi dor, ciel Excedente de1 productor y del


Excedente Total. (2 puntos)
b) Suponga que el gobierno establece L1n impuesto de US $
60 por equipo de sonido " ¿Cuánto pagarán ahora los
I
consumidores? ¿cuánto recibirán ahora los productores?
fncluya estos datos en el gráfico de a) . (2 puntos)
I

I
c) ¿cuánto es la incidencia del impuesto que asumen los
I
consumidores y los prcductores? ¿euién asume 1a mayor
parte deI impuesto y expliqué brevemente por qué?
(1 punto)
d) ¿cuánto es el valor def nuevo Excedente del consumidor
y eI de1 nuevo Excedente del productor? ¿Cuánto serán
1os inqresos fiscafes? (1 punto)
e) ¿A cuánto asciende eI valor de1 nuevo Excedente Total
de Ia sociedad? ¿A cuánto asciende la pérdida del
bienestar o eI coste de eficiencia? Muestre esto
último en eI grafico de a) (1 punLo)
B . La st cuien-Le tab-La nuest_r¿i cuatro
bienesXeypar:aun ccmbinaci ones Ce ios
cletermj naCo ccnsumidor :tepresentativo :
Ccrnbrnacrón tr. ^^^-
U!U1IC¡ \-ana rant . Cant Canr. Cant Cant.
X 2 3
A 4 5 6 1
Y i2 6 4"5 3.5 3 2.1
X 3 4 5
B 6 1 B
Y 72 6.3
8 5 4.4 4
C
X Jtr 5.5 6 1 B 9
Y L2 o 7.8 6.1 6 5.4
X 8 o
D 10 11 L2
Y t2 Qtr. 7 6.3 5.7 5"3
a) Grafique ias culrvas ce indifer:encia
co¡nbinaci cnes A, B, C y D en un nisrno de r as
s-. st_-ma cle ejes
Y consigne correctamente Ios
b) Determine r-a Rl'{s entr e rccos ",ufios puntos consecutivos
iie ras cuar.-ro cur:vas de rndife.encia ({-rse
aYuda aba¡o rnostrada) . tabia de
(1
", ;)!Yo":1.lfo q". p, : s/" z, pv : -q,1. 1 y ta Renrapunto)
(R)
/ . r o a ! d1a (todc lo c;asra tocio en ia aclquisicién
cie
--s d:s ¡re::es); trace án et gráfico ;; ;;-í;-"""a" o"
Presupuesto -\., consi gne I cs val o::es en ios
extl:er,cs
lsinismo, determine y consigne r-as caatidades de "

bienes paf a r as cuares er consuinido-r maximiza


ui-ilidad o sa:isfacción LctaL (ópLimo) _ su
(1 punLo)
d) S, el !x
D urrl smlnuye ie Si.
2 a S/. -!, o.ro.r,..,iendc
ccnstante el P, ios gusrcq |¡
consumioor; trace en eI gráfico ),de a) Renta la
(R) .lel
consigrne 10s va,r or:es e, los extremos. nu.eva Rp y
Asimismo,
l:l"t*:i" y consisne las canridades ;;r. ffi"
sr.ualr-on en la que eI co'sumi-dor max_imiza
o satisfacción total. su utiticiad
(1 punto)
se sugiere que para d.esarrollar b)
ayuda gue se encuentra 1íneas áhajo: utílice 1a tabra de

Combinación Combinac- B Combinac.


A
C Combinac" D
x Y RMS x Y x
¿ 72
RMS Y RMS x Y RMS
3 L2 5 L2 7 12
3 6 I
4 8 5.5 9 I 8.5
4 4.5 5 6.3 6 7.8 9 7
5 3.5 6 5 1 6.7 i-0 6.3
5 3 7 4.4 B 6 Ll_ 5.7
7 2.7 I 4 5.4
9 12 5.3
Piuru
{Jruiversid¡td de
Fhcultad de Ingeníeríu

cwRso: ECoNoMÍa- SCO.


SECCIÓN \\A//
EXAI{EN PARCTAL
PruRA, MrÉRCOLES, OZ
APELLTDOS Y NOMBREs:. .ih

La presente práctica tiene una duración d.e 3 horas.


E1 puntaje de cada pregunta. se encuentra espeeificado en cada una
de e11as.
No puede tener apuntes de cIase, fórmulas, ni otros materiales
sobre el tema. sE ACEPTA EL uso DE cArcuLADoRA.

r. RESPONDA. CONSIGI{E O }4ARQUE, DE SER EL CASO, Sü RESPUESTA A}ITE


LAS SIGUIENTES PREGUNTAS:

. indique có¡rio se Cespiazará l-a cuiv¿i ie cienan a clei i:iercaic


'ce un bien noriitai (cielecha-D, izqLr ie rcia-r, no se ciespl aza_
\iC, rncj-^Le:min¡-ido-Ind-) sr: (1 punto)
,/\ a) E1 piecio Ce los bienes srtsr_j-t.r,tros b¿-r-ta. ( : )
i ,.\
i .'\'{ bi La nc¡b:ación árlñrint:,. ( : 'i
.'\ t i,
c) I.os EL.istos iel púbiico se cirr-igen hacra ctLos b.renes.
\. \-' (
i\L, )...
d) El prec-ic ce los bienes coincleriertLlrrios baja. (.
e) Se .jislribución -_) i,
de ia :eni-a se hace más equit-arLÉ"
(- - -"") ;
f ) La renta naciorrai aument-a . ( ,-_ )'\_.'
g.) Los bienes complementar:ios se encarecen. ( !. -) -t.
h) Los b,..enes sust t t-utos se encalre cen. ( ,i ).-,"1"
. Corsidere el caso de un consum-rc1ci: que aclquiere dos bienes
(x e y), los cuales presentan curvas de demanila
decrecrentes . rndique, e o cada urra de i as sitr:aciones que
se planrean más abal o, de qL1é trpos cie b_ienes se trata.
(1- punt'o)
a) tsien i nferior "
b) Bien de primer.a necesiciaij.
c) Bien ie lujo.
d) Brenes i ndependientes.
e) Bienes complementar.i os.
f) Bienes su,stitutos.
g) N-rrrguno de los anterlüjtes.
un aumento en la renta del consumj-dor desplaza la curva de
demanda del bien x hacia ia izquierda. t' (^ ) !.'
un aumento de1 pr:ecio de1 bien )!, deja inaiieracla ,.la
cantidad demandada del bien y. v. ( ,|i"i i

Ante un aumento cle renta, La cantidacl demandada de_L r)].efl/ Y


aum.eirta en una propcrción maycr quc la lenl_a.. (i,ó
J'' ' i l ' 'l
I
un
dei bien xt provcca
rl
Un aumentcr''')e I^r el Prec rO
I ¡ cerecrl a cie I a cur-va
'cle dena¡da del
¡- a'
,-te sP a z a'r''; :l Í- ita': ( -¡ )i-r
\\i -:-

;i.; y- ?*'t o ¿,lt


I
DY
bienes Presenta una
)\.
-u
ambos
i\,^r.\\..
\ i
La artt,u C'e cremanda
penclien-ue negat iva '
cruzada de
J-)-'
valor únlco de elasrjcidao (0'5 Para Loda
Puntos)
. ¿Verdadero o f also?
? Se
una
--l cr e : -=: --=- I
clet *:-':- - -a- ii rie
cel,a:-l'a es cl: sr-:,1 a 2 ' '.-:--r
r\ o fal-so? (0.5 Puntos)
,'
\
l
k,'
;
nrrnro de Ia
:
mj
,l ¡ ,

br\ --Í"c"utrCo en i-ln pals


tutl-ente ']:tul
4 . Un arlálisls
ecc-nómic: .
ru:.'='"1"(]-ñttt
'
'Jo*spc)ricienr'es izacas scbre el valor:
euiopeo/ recoge a ciir¡ersos bi enes:
cre árQUflás eiasirci.r.¿.= t"= siguientes ciar-os
In ccnc rei.c se-
tl ser'vic:-os ' ' -1:.::.:"-:
e iast . c,,-raá-p;;.,;, iá;r"'.-¿::::;
-l' t a e L-UL*rurg**
cl¿ , :'- ;ii: ;",i."ij l:
(1- punto)
Elasricrdac-pre--rc
.slall.lleilLC -e:
' nÓm'rco de estos valores?
ál ¿cuár es
¿CuáL e s r:. signi-ii(jatru
r:i riq"ri.l.?:-" :::,.,,.
E!u1rJ!"*--'
.oara r.--.!'i ii'-::- ¡:?
¡r=.iii,:ar,o-.,?
se
t"$ =" t".; l::o:",
áeué ..srr.'u.,..,= :ftj::;:.1i;=*' .=- -,i--=-."= r,

b) ;, # :-.. t ; . _¡ .; ¡r" ::: = ;:'.' : l' ?:. ;;;1. t;fi : :


"'
",!:#],li,l
", ;""="
,, I 3 I' I i : ": : : : : " 3 5
='
",' l :, : i I i :.'
"="'Jffi

\D i'l i:i
f\rY
¿\

'lor se encuentra en
Ia respuesta correcta' Un consuma(
',r,-.-rutto"
5. M.^rr.;1le -
á'
equil-ibSio .:'.t::tJ"? ::
l,p^Ti.:-;.t':
podlro p)ruu (1
;;r";-="lrsfaccj-ón-
nayor sdLrsrauv ^l.
l -.to'--¡
-¿0."u
. ." \l punto)

iui =ia".ción de equrlibr r¡;rrreniéni constanter s los
reanteniéncJose
\jil Un aumento en La renta
t precios. _r^^ cle
r -. rLargo .le la recta de presupuesto
I a r€
"t,l
b) Un movimiento a lo
oto;;
i:: misma proporcrón'
c) aumento cre ::":]':.;¿-t.
la -r'-enta
manteniéndose constant'e
"

d) Iodas Ias anteriorcs '


e) Ninguna de las anteriores "
ExpJrque por qué en det er,minado nr.olir--nto / iln rtici:ei:eri':o oai
la renLa ITiot-r eraria puede no d¡:sp, a7-ar - ¿l llrlcta .]e
presupuesto hacr'a la derecha, e inc- irs.-)/ p;eie tiasIad..^::,ra
hacia 1a izqurerda. (l- punto)

ñ
l1
§\{
ir \\ ,,

"""""""""""'"ll""r"

""""""""""""""i"'t t'
..,'......'..........,.'...!.... t
I
\t:"

7. rylqjg!-- ia j_ternativa correcta. Consi cie re eL caso cre ln


a
CCnSuntCOr que creSt,na tOda Su reni-a al Consumo de
orcCuctcs, a y c. D.t-cho cci:sunLidcr aciquiere actLiaLnente t,ll)a
Canl,rdaC tal de los dos brenes que Se C:UIrL.; Le q'le iá
utrlrdai m,a i:qrnal que Le ::e ,oorta ia úitim.a unid.rcl ccns-l::. i, \")ttg .'

clel L¡ien e es e1 dobte Ce ia uti irciad ri'ar:ginai cleL bien c "


^
lc
LT sumi clci: estará en equrLib::"lo s¡ :
L- JIi (1 punto) :

a). ,
Ei prec: c oe a es '¿ del precio de c.
\r\ \\/l) E,l puecrc c1e á es el- Coble ciel p-rec,o cle c. arú"
V\ .b)
c) Ambos tienen el misnc r-.llecio.
,u5^
d) E-i. p::ecic cie c es ei clcble de-L pr:ecio Ce a ?" "
u\"
e) i\irigrrna cie las ante¡'iores. r)
'f a- I

/:) I
B Sobr e La f un: ron cie ;c¡'ui't:--c -med:c y itroductr; marqi-tta 1, Y" D!
rr ir .r
.,.: n loa--a
¿_\.r,yo-1,. lac - at son ..¡e¡d-a.jeras (r":) r 'lA
(1" Punto) U?u'
ia.lsas (F) . \ __-2
a) i:r ei opi-,7trta iécnicc, el proCuctc rrLedio es máxiino- ('¡ )
b) En ei rná-t<. ¡no técntco, eI pL:oducto me cl io es mavoi- qLie
áq\ el Prociur:l-o marginal . ( .: )
.¡\' c) Err eL ópttmo técnico, se iguaian el pr:oducto ineCior -v
el proclucto marginal. ( .,' )
d) En e1 má:<imo técnico, el protlucto margina I es mintn.r.o .

, ,i,
Indique a cuáles cle las siguienr-es f,rnciones nos estamos
r:efi:.j endo más abajo: (1 Punto)
a) F.ect-a rsocoste.
b) Función de coste marginal.
c) luncrón ie coste total medro.

s d)
e)
f)
Función cle coste totaI.
Fuilcrón de coste variable nedio
ilrnguna de las anterior:es.
Rel ac j ona 1a canticiacl de producto con el m.inim.o coste. de
prcducc,or. (- )
Disminuy-- en tanto es superior al CMs y es creciente cuando
es rnferior o. CN ,. ( C. )
Muestra e1 r:itnc al que varían los costes de produccrÓn. .
(b l/
Contiene las combi naciones de factores que la empl:esa pueoé
aciquirrr, daclo el coste y Ios preclos de los factores.i--."lll
(rr r-¡r-rrtn mí n i r¡¡ s.i crenoini na óptimo
"t-",,', de expTctación. ( :iL_)
3r
10. Una empresa en Competenqiá Perfecta iebe CERRAR.
Tlllut;,rcr).HlivtEi..i iE (ci,) si
de1 rr.ercado (LP) cuando......',,--f..: LI II-L V_I
,^\ de producción maximizador ¡ide bene¡icios ce üla. encresa en
(üi1 Competencia Perfecta sucede
\
(1 punto)

It. OSSAIU{OLLS y/O elut¡¡lQurE, tNfeRpRErU Oe SeA UL CaSO, v/O


CONSIGNE SU RESPUESTA ANTE I,AS SIGUIENTES SITUACIONES:

11. ics gus-Los Ce un cc:lsun j dor se col-o l,ri- po r a iunc'ó¡-


c]e u:riioad sr guien-Le : U = (2X + 14) (5Y + Aslrtz
siendc su Renta Monetari.a (R) Ce 224 uniia,jes rnoneiali a-s,
,ol precio Cel bien x eS oe 4 unrclaaes ir,,u.::3l ar:ias 1,- :
precio ciel bien y, de 6 unicacre-s Ilior.te -;arias. A] rescecl-o
9'' c-'-?-'r' :r'::-e' I''l'á:
a) La s i-r-uación de equilibrio de GSI:-e consum,rco-a "

t/ (1 punto)
¿l\ b) Si el pr:ecic ciel br y se no':l-rf ¡ ca pasai:<1o a ser '¡
"n
unicaCes ;Lcneta:: I as, ¿ caicuie y il=:er:ir' j re ra nueva
s -l-uacr ón ci= eciui Libri o ? ( 1- punto)
\\,
c) L,i e.f ecto sust¿ t-ucíón (trS) , erec-¡c-' .eiLa (EFi, -'"

\,\ e-fecto tctai (ET). (1- punto)


d) lln ir:nc-rórl a1 efecto tota,r- (1T) ;; c=l i:iecio i3n'a
(El¡?.) , rea,'Li.ce el análi s: s resoec']:c Ce ics bj-er=s '{ e f
t;il ccmc se :eal rzó en clase. {1- punto}
e) En -r-.1--pg-!o-qiqlrce i¡.uestl:e ias sirrracjc¡nes a), b) y
c) . (J- Punt'o)

t2" La func:ón ce costes variables nedios de una er,presa


es: CVMe : (X/5) + 2, clonCe X = cantidad de producción"
Calcular, eL ó¡:t:it,t .le e::pi otaci ót, sabi enCc Que , para
5 s:e total m.eclro (CTi"1e)
"b-,..ner "
es de 7 untcaoes mone''Lari as (2 puntos)

13. Una empresa de servicios iedicada a la Limpieza oe


ofjcinas, actúa como precro-aceptante en el mercado. El
pr:ecio que plrevalece en el- mercado es de US $ 30 por hora.
Sus costos totales están dadcs por I a srguiente funcrÓn:
CT = 0.1q2'*1gn + 50. Doncle q cantidad de horas que -a
empresa oeciOe laborar po¡ semana. Calcule y deterni ne:
a) ¿qÉLr-_ee horas optar:á Ia empi.esa por laborar a ';
semana pal,a maximizar ganancias? (1 punto)
b) _C¡-LSl¿g ias ganancias máxrrnas de la empresa po:
semana. (1 Punto)
c) Grafique estos resultacros Y muestre en el mismc ia
cur:va de oferta oe la empresa" (L punto)
U|\IIVEr:ISIEAI) trIE T;IIUFIA

ASIGNATURA
TRABAJO No ,fh rcral
NOMBRE
FECHA L)¿/lc,/¿¿¡ t,

trl
tr t--)
FrRMA oel e{orlson ) AYUDANTE CALIFICACION RECTIFICACION FIRMA DEL PBOFESOR

) i)n. trx
ii (2x+r',)l (sy * ts)'12 r
j W--t rt )H 6-1'/'-J"'
l
)
Uy -- Z(Sy + tsSt/z
i)- -
¿- (2v + t1 l¿ (f U tts)''j
t.}"
-l
r.^ -i)rr \
l'2N
\/" ).1 ts)
I -¿
i/ - t
t)/ i i .t /,.. 2 (1 ,i -1't5 )'' {{-
v -.\ /
f]t - ¡- ,,
\-v
I
',iY
1t
'
7l(rltY.¡ 1

tCI
'z ¡:.,'at,'i' .)
41*7.x-tl
{)*..x tfy ,7= R,
r 7r.--
' 1Y
--__--
+t
_-
--,x
¡
4y + 6y -- ZL4
+(3Y +z) tLy =zz4
4Zt¡ +i+t'\--?2+
4l - 2¡6
,-

[ \ -tLt,
tr-_
tl ,
¡) U, - *':*,
7x
--)
7(5t+t5)'-
rly fly' -_--l--
S(Zx+tq)
- {t/y_r,A)
_ / ^\ )Y'24=
r
zx+tl
2(5\tts)ill É(2x+tq)- 3q tlv 't +|L--'x++
Px X +Pv y=r¿
\/.,' +s -
74, i-,
4x +8\ =zL+
+(4\r5) +fy=zZF
4¿y t zo {8\ --zL4
1a
I

r r'l'l Y

e)

¡)
1'.
\
"t

I
C)
C\)
br¿vr{ " lliqu u**{-
J
ht*'n'.1, -
i*^r§"r
|,,*
--

\" )
I
i

\
\ -
Cf=
^'+/-x+ZC
=
CHc,= 2x +¿
-5
Y -5
-=1x-)
'r
i^) (i -^
<-ru-t
va
/\J
- X
/i
)
i/-' \ l{.,:;;7*
/
-'r e)
-j" /- -l - ,1, ! ,',,.' .- t. o'L + 5 c)
,'- ]
tq) CYc¡ -- ( rir¿
O.21'r r ) {; ,::- o. L q't il^,<--.
--r- ( \-/'f .l_;---r
1-
,,
tl\t
ll .v
O-2 r.,2 ¡ t,¿{ = c. L7- * t, +"YJ.v \
')i\
l-,/-1 \
r\ Lt
l¡ /\
\
-1 -- 5:r-
a-L c,z I \l \ \v
'/
\ll
.-r
{l
= L'¿. ak: -A(1 7)atn-a.,.a---..
t

(^)
cl--O.t (21 ,1cu )-+ to( 22.36) l5c':

C rn.3
,:--

Universidad de Piura
33
Facultad de Ingeniería

c:uRSo: EcoNoMÍA- Eco.


SECCIóN ..A'
EXA}4EN PARCIAL
PIURA, IÍARTES , 06 DE .OCTUBRE -DE 2OL5; 08:00 Alf . .i.-. * , ,n,,
ApELLrDos y NolÁni;, . lvió1 r"i* .üirÑ, ol. -'?.9'!:".7.
-fx\t\#; .

La presente práctica tiene una duración de TRES (3) horas.


EJ- puntaje de cada pregrunta se encuentra específicado en cada una
de e11as.
No puede tener apuntes de clase, fórmu1as, Di otros material.es
sobre e1 tema. SE ACEPTA EL USO DE CAICUI.ADOR;A"

f. RESPOT{DA, CONSTGNE O }nRQUE, DE SER EL CASO/ SU RESPUESTA ANrE


I.AS SIGUIENTES PREGI]NTAS :

1. Considerando que eI valor de la ei.asticidaC cruzada entre


eI precio del bien llamado x y la demancrada del bien
llamado Z es de -2.8 Se sol-: cita que usted consigne Ia
.",---*-ion' económíca de este valor si se estima que el
i -rornral¡¡'

pte rementará de S/. 75.20 el paquete


de,tr25A qrs. a S/. 16"35 en la ntacíón. (1 p)

2. Al realizar el estudio de la efasticiaad renta de las


personas del nivel socioeconémico C en Lima Metropolitana
durante el mes de mayo del año 20L5l respecto de Ia demanda
del Bien denominado R era se detectó que ésta era de -1.9
Considerando esta información y que por efectos de la
desaceleración de la econornia, se estiman que ios ingresos
de las personas que conforman el nivel socio-económico C en
dicha zona geográfica, disminuirán en promedio de S/.
3,422.00 mensuales a S/. ?*gZO.00 mensuales, se pide que
analice dicha situación y consigne e1 resultado del mismo.
Además, indi-que ¿cómo se puede explicar eI comportamiento
de.1 en R que usted establecerá? (1 p)

1
\

3 La incidencia de un subsidio beneficia más a aquelLos que


trenen-aCurVamasffin.....odichodeotraforma,e[
aquellos que tíenen la curva menos ..+)ílÑs:i......... De otro J-ado,
la incidencia de un impuesto recae menos er aqu=-los que
t ienen la curva "*:.-.......1ncÑHffiu..+/-.. y-recae más e-n- aguellos

v ñ,,é
Yqe
turu!!'i o16¡ Ia luru^

de un impuesto en un pais
.inraá-.-.tú*Iád+9.........
(1 p)

4. La aplicación ^3ñóra
l:c

siguientes co^nsecuencf as: a^)^ ^1,.* actividad conomlca


disminuye. nl ..-tL..p.tss*q qs{* #+*t.ql- PH}l$*
..*J-. ...P.st^^,. E ** .Qs§+... *f. p.rs -d\+s.\o- (... ..Ql¡¿¡,rm*ct'
. . E1
Estad'o recáudat imrpuestós y e) tie e-l'lü3\^c-ta-
\\tt./t
Atl-QJa.*'t§§."...
recaudacion. de impuestos
Sin embargo, la recaudación +mpues'
,/
E cr ¡-rn
LJLOUV ^eimite
V fa,fnr,üi.§§+p..(..(rt.*=U{,(»rr-..,.JC.1u...
\ I
ffii.,',.'rt;g,¡g'
nosotros conocemos como equidad. (1 p)

5 Compe -r a Perfecta ebe CERRAR


TEMPORALMENTE (CP) si Ous
.........F..x.u¡...{:.ts.t1.91..9*9....!.-v.n-e- y ciebe sALIR
del mercado (Lp) cuando......f.ll-tf**...*.:...if.:.r*-i..3.1*-..f.$.t-+-¿.... EI niveL
L1UllttCrUou9\!L/Uuq]}uv......j.....i.......'....x.
de producción en el que se maximízan los benef.!g:-os de una
,§ empresa en Competencia
-^ññ^+^-^.i - Perfecta
D^-€a¡l sucede
¡ar-la ¡¡r:nr'ln
cuancio .. J--\--. l\ l1 n\

En estos últimos días nos hemos enterado de enfrentamientos


entre pobladores y miembros de Ia Policia Nacional del Perú
por eI proyecto minero Las Bambas. Dado su interés
permanente de nuestra realidad regional, nacional y
mundial, coment-e qué conoce de aq'é n-n.¡a^tO.
(1 p)

v
rr. eiuunrQurg, rNrgRPREtg ou ;En sr, cAso, y/o
orsanuror,r,n v/o
CONSIGNE SU RESPT'ESTA AI{TE I,AS SIGUIENTES SITUACIONES:

7. El úItimo fin de semana, el MINCETUR hízo público los datos


sobre la temporada turística del periodo 20L0-20L4 " Según
1as estimaciones, las principales ciudades visitadas en el
Perú fueron Cusco, Arequipa y Ancash. Los economistas del
centro de investigaciones de Ia Universidad de Piura - UDEP
han calculado la demanda y oferta turistica de estas
ciudades, obteniendo como resultado las siquientes
ecuacaones:
Qd .r"."rr". = 4t 875 r 000 ' 25P
Q¿ ¡r:-tá'ri"os = 8'0501000 - 70P
Qd otto. palses = 6'500 r 000 - 50P

Q" cusco = 6'9501000 + 10P


Q" Arequipa = L' 170 r 000 + 2P
Q"

Donde P precio promedio de Ios operaáores turisticos en


)</ / . Al
l1I Iracna-L^
C¡_UCv LU 5C ^'l ^-' .
-t-rIL-Le
a) Calcule y determine la denranda , total y Ia oferta
totaL de turistas en el Perú en eI período estudiado.
Asimismo, la demanda y oferta Por ciudad y su
di=tribr¿ión porcentual. s p) (1 .
b) Calcute y détermine la de¡nanda por nacionalidad asi
como su distribución porcentuaJ.. (o.s p)
c) E:time el número total de turistas si el precio
fuera de S/ . 70,000 y calcule La eJ-asticidad de Ia
demanda total así como Ia e]-asticidad de 1a demanda
por nacional.idad. Interprete }os resultados obtenidos.
(1.5 p)
d) Si los operadores turisticos de las tres ciudades,
aumentaran sus precios eI año siguiente, ¿qué se
oqnar: del sasto total de los turistas? ¿Será igual el
uJyv- q

impacto deL increnenco deI precio en eL gasto


turistico cie Ios brrtánicos, alemanes y de otras
nacionalidades? Exptique Ia razÓn. (1.5 p)
e) Calcule y detqrmine la e1asticidad de J.a oferta
total. asi como 1a eJ-asticidad de Ia oferta por
ciudades. Asimismo analice y sustente ¿en qué ciudad
aumentará porcentualmente más Ia oferta de plazas
turisticas como consecuencia del futuro aumento de los
precios por parte de 1os operadores turisticos?(1.5 p)
8. Daciala siguiente funciÓn de costo CT = 1,000 + 250X - 5X2 +
0.5x3 donde CT es el Costo Total y X es Ia cantidad
producida. Para facilitar sus respuestas de Ia primera
parte del apartado c) debe llenar sus datos en el cuadrc
¡.r rnrn I í n^-^ */,^ -l-^i
juoJ ^
v.
oJ,J u---L --r,ts'*i -ltc.i.>
a) Derive las funciones de costo total medio/ costo
variable medio, costo fijo medio y costo marginal-
(0.5 p)
b) Calcule, para todas las f unciones antes seña-ladas,
Ios valores que adoptan para las cantidades discretas
producidas entre X 1 y X 15. Además, obtenga las
cantidades que corresponden al minimo det costo
variable medio y del costo marginal (2 p) "

c) En un par de e¡es cartesianos, grafique las curvas


de todos los costos. En la primera gráfica deberán ir
1os costos totales mientras que en 1a segunqe l-os
3
costos medios y narginales. (1.5 p)
d) Analice y exponga Las relaciones entre los distintos
tipos de funciones de costos que usted a encontraCo en
e1 preserrte caso/ considerando 1a teoria expuesta en
clase (1.s p)

Cantidad CT CV CF CTMe CVlle CEIIe cMg


Producida
X=1 [Lq5., E [\5 rB
-Tqiü_.l iooo tt\5r 5 115,5 (ooo §Yt,,5
X=2 qlq 1()()§ { r\L A\L \r'1l)- ot,ó \c
X=3 Ft rt' 5 ,1lt,r',' \or)§ q4.?,t ¿11§ 33i,3 a3i, E
X=4 lq5 \64- ' i ISOO He8 ala'z Aa§ L3\
X=5 "-
u r?)5 \\ ,6
it r00O Llj+,, 8
{a>,'5 )so f,i\5
[= 6 eqt-\ tW3 \o0 0 ¿loq,y )q? /tL,{. T\
X=7 arl(,5 lL'{Gr5 I o0'c 3tllY raq,5 i\t{ tt, ?_51,5
X=8 tatÉ, 1q'}b \OOO Xtv q!- t2.5
&. uLG C"'

[=9 jrolrS I3sl1É \ooo/ 1q6,ü (iqh.É itt Lt\r 5


X=10 lqo o L6oo , \oo,6 '.1q., ?.50 ioo 3oo
X=11 3E\O\5 )S\orÉ io( § lQbr\ L6or,5 Q,O,9r \l-l '5
X=7-2 Ll, \rtr{ arqü (oo O a\ci, 3 A.G¿ {'¡.3 3r16
X=13 .t5o =. ,5 1':o1.S io00 3.tel q )6q,5 1G'r 1?- á{i, b
X=14 \qqL -f-\§.\O,-r
l"¿r rOcO nJq, \ a.1 I 't I ,'tt3
qc; 6+
40q
q3),5
X=15 53\{,6 qq it\5 i§cr¡ l6LrJ\1 Iq},5

9. Un est-udio efectuado por eI centro de investigación cle la


UDEP a una microempresa piurana que opera en un mercado
competitivo, se encontró que su función de Coste Total fue
CT = Q3/3 2Q2 + 6Q + 1,000. ¿Cuál será el precio más b,ajo
al cuaL la microempresa estará dispuesta a vender una
cantidad positiva de producto en el corto plazo. (2 p)
R
UI\IIVEFISItrtAtrt DE PIUFIA

TRABAJ. No ?^r*;*
FECHA 41rc/ ry

FIRMA DEL PR O AYUDANTE CALIFICACION RECTIFICACION FIRMA DEL PROFESOR

a) A s,a^^.L .t/t wL "ln; ,fu ¡,L-a.¿ÍM,araLt


daa,P Tani ',xL ,I-14f-{ ?fu*
oülrmarwdfu) +1,Y
4,,-
'WrLoúe;p14t/J,
ry ¿t/,^ .0r2L ú y'e rua.¿.<dad-.
'l
Y* ¿uaú;p!, dt u\.rt o¡wto-a,ur Añnu? ilartls *t P,r,a't -,h'
tpl¡toffiiiz-u'V ,¿" 'ry. w/r-
I h^/to^/ üÜrw 2 tr*

ry f ?ru )4, ¿;X,Lrt",fu rys f,r-t"n *l


,4,t' lraaran /*1, ry
,1011_
W -10¿' l¡onAA¡No' /W 3
"[^ dhrrrr;d^ 'nn era,"n'trtaTÜ/
le) U : [ex + ru) ¡rY + m)'h
,\-,

d) 6y+ 6)'l*
r tr) 'lr t [2xt lq)
N- fd [o¡+rq)T
j
t5Y

& ld,
/to (5Y +t5 )'lr= (ax+14) L6 \ 5)tt'
(px r l¿r) dv $l"rV+
ü =ld bv{¡'r\1 Vvrrli»
dv Ld,y( )

_?* _\
p [5,¡ + ti ALsY+É) : E
6
ryt- -/
5 Llx *l Alax+ lq)
0lt1l T,
t Líy +ts) Y+ b, = J^
b
Lt-v rt4)
?*.x+ Qv"l=K üY+tY = e,x+ 14

¿rq + bY =l?q 6V+U 2Y

(a)t ¡) tt) = 3?¿t


ñu x
Q
x' b1

rú) *lJrrtu
lY' '-

u) D--,^,*'* d, !^ Wúrru¿ ,lyrwa <.

yg$ i 4-g {x t-8) = cpq


,"T l-ltl rr[rrY +6) { lY =

fY + *t4," 2..x + tq

\y+5 = x

qh@ca""^""r" í
rl3 u(q) + o6)" eu3

TN+Ey = aqt
rY_' il,J f
Qttry +5) +

\Y_q,,
Y

{!! 6L t,l- )2-4 b1,b


e\)
,^
l

q,{"
\\
,\ri
nq \
'',

I
I

1 V,-)
I

/
^r
t,l le
r^v ql -
.!r= tp - Cfo
e-L
i \\-*-
'--tr'-* QTt
¡ \-'.
''\
] 1,
L
*,---
r- t -\.on'
dt -
a ?s :,'r$
47:-/ -> C
'!)
{-*: €-g
ü E-t: dl (ar,r3)
"(e,q,i)
.E-0 L d. I t z, e.s) w ( rq, g.5
E- T O.

d,t- ( Er, ra) aL ¡q t s.b)

Jrt * * t^/n J¡rrn .n*núal-


^ 5 il- ,hu^, y úb wk ¡wr,
, ..u, wvl
)

'.8- R'
x dL 0Pwu¿wunU tW = ru¡wtnuU
?rv'tt¿tvYv
,l ,[nÁ Y iyf..lro
W Ab 4 y'l,- ü.n)"rñ \rü f,f Jru,* x
. t,.*
V ?q'u, vnat, Jnadfrl K&
nw ,*,"T Ld.Vt' ftn n¡¡»:ra,
Y ú-'wwyqAir -y ,lq efuVU

W\,rW dt
$y*r* l,);,,,*!,,o
lAL . -
la w,,,*, . Lln;^, & aaa.*"
/tL tt<¡ql*&wd".

&- a*g,vYnidfrl ,/)¿ dá wAL Ü ou ,x;h'dAa


q UJ\A dL (Wqt t*rY"
{-
,.'-,


o

Yt
*,,a-
@ r* 5, q\
1* q4tí, ,,tr4" "/A" onh,.rna, eT ,!
"¡,fu .A a- d,,;,,Y
'ttN bA
@twwiolú, f* -tirltt, /-, +ná¿.rt* A,ürhfu.dl I W
,wd;4,.¡tyt i., n
ln,*vna etfrr.,rrA,r¿ -{*

0 q, = w(s- P) - fo -to?:
= (5o- rc?*) U+P*y-'

d&
ó+

Lw"
?f

I q=go- top") 1
\
5*?*

(j+ ?.), + Lso-to?*lld==O*


Lde
-ñq
/§" : -o(s+p*)-' 4(ro- lo P*)(D @+ ?-)-' ,'(,)
W
*6o* o?*
A 25 + to?* + P"z

)€p- {-ao- pP-


\ -_-----_:- @+
\az**e*). & =
D bPo +

'b
ep '(§vs
Universidod de Piuril
Facultud de Ingeniería
CITJRSO: ECONOMÍA- ECO.
sECCIóN .¡1"
Lafs(
EXAMEN PARCIAI
PIURA, ITARTES , O7 DE OCTITBRE DE 2OL3; 08:00 AI'f .
APELLTDOS y NoMBRE s: . Tfrfu . . *l@<r:t. . .p.ü/N'JN. . . @tU.t*r+

La presente práctica tiene una duración de 3 troras.


EI puntaje de cada pregunta se encuentra especificado en cada una
de e].Ias.
No puede tener apuntes de clase, fórmuIas, ni otros materiales
sobre eI tema. sE ACEPTA EL uso DE cALCIrr,ADoR;a.

I. RESPONDA. CONSIGNE O },ÍARQUE, DE SER EL CASO, SU P,ESPUESTA AIITE


LAS SIGUIENTES PREGUNTAS :

L. rndrque cómo se desplazará la curva de demanda del mercado


de un .bre¡ narmal (derecha-D, ízqurerda-I,L I no JUse UUJVJU¿(tr
despl aza-
t.)) si:
NO. (1 p)
p) ,
a) Los breres susl- Luros se eñCqr+cen. ( > ).,
b)
u, Los rlrtPuc5
luJ impuestos aumentan.
LU5 du1tle1]Ldll. (( f\:.J
h!, ,)'**-
-*-
c)
a\ La población en el
T: nnl¡l ¡ v; A^ ^l mercado del bien
ñ^-^-.!^ .l^-r Ll ^- aumenta. ( D )
, { \

d) Por cuestiones sociales, los gustos y preferencias oel


público se dirrgen hacia otros bienes. ( 'r ).
ofq e)
f)
g)
El precio de los bienes complementarios baja.
Los bienes complementarios se encarecen.
El precio de los bienes susr,itutos ba j a.
(x. I
(r)
(r)) /.,
Y
h) El poder adquisitivo de las personas ha mej orado.
('P ) 7
suponiendo que el .".a,or ce ra siguiente elasticidad
e cr¡-¡zy)t : -2.3 En ese ccntexto, consigne a conttnuación la
interpretación económica de este rralor si se estima que el
precio del ,b:en X se i ncrementará de S/ . 5. B0 eL kg. a S/ .
6 ."20 el kg.

0r{
3. Considere el caso de un consumidor que aclquiere dos bienes
(,X e Y), los cuales presentan curvas de dernanda
ciecrecientes. rndique, en cacia una cie las situaciones abajo
descritas de q'üé tipos r,le bienes se trata. (1 p)
a) tsien Irif e;, ror.
b) Bien NcrnLal oe pr-nera necesidad.
c) Br en Norma : de -uj o.
d) Biene-s _ndepe:-dtenres.
e) B'r enes conoletnenaartros.
f) Btenes susr,turos.
g) Ningrno ce ios anterio:-es.
\
El valor de la elasticidad-renta es positiva y
Un aumento en la renta del consumidor desplaza la curva de
demanda del bien X hacia la izquierda. ( tu¡
Un aumento deI precio deI bien Xt deja inalterada rd. l_

cantidad demandada def bien Y. ( d,¡


La elasticidad cruzada de ambos bienes presenta un valor
negativo. ( L')
Ante un aumento de renta, La cantidad demandada del bien Y
aumenta en una proporción mayor que Ia renta. (t )
Un aumento en eI precio del bien X, provoca
desplazamiento hacia la derecha de la curva de demanda del
bien Y. t[l
4. En una curva de demanda Lineal de pendiente negativa , LLa
elasticidad-precio de la demanda ^ri^^^
Iruusu
l-^ñ^r
u!rrur un valor únicc
para toda curva de deman ? ; Porqué? !o-5 pl
0fl
De otro Lado/ en una curva cre demanda lineal- y vertical, ¿la
elasticidad-orecio de eman Lo en
cad to de la Porqué ? (o .s p)
oq
5. Un anállsrs económico efectuado en un departar:Lento cie ?erú,
arroj a que Ia Elasticidad-precro del serviclo oe agua
potabLe -A.21 y los servicios profesionales cie un contador
-1.35 Considerando estos datos, se le solicita responda lo
qin¡rion i o. (1 p)
a) . r],.Á
¿Vuu argumentos se pueden emplear pa r:a
justificarlos?
b) ¿Por que se drce que a Los consumidores les :nteresa
?,( que los productos que adquieren en ei mercado tengan
valores altos de elasticidad-pi:eci o Y, sin embargo, a
los productores les interesa que tengan valores bajc-rs?
6. JUru
c:. 1^ !-ro! .1uE la
\r--^,.^ respuesta correcta. Un consun',idor con
prefe.encras usuales respecto cle dos bienes, X e 'l , que está
incirferente entre las siguientes combinaciones: (X,Y)
(4 , 6) , (X, Y) (B ,2) . Indique cuáI de las siguientes
combinaciones PUEDE ser prefe;:ida a las ani,eriores (1 p)
X,Y '-,i)
^)l
b{. ','-
,'l .t , -
'c) (X,Y) (9,1)
7 ExpLique por qré eí- le'-ermÍnado momenLo, un incremenco de Ia
renta monetaria puede mantener: constante la Recta de
Presupuesto (RP) o desplazarla hacia la izquierda. (1 p)
........(e...........
...f/¡f,*{,"".....

8. La convexidad de las curvas de indiferencia de un consumidor


entre dos bienes, X e Y, implica: Marque Ia alternativa
correcta. (1 p)
a) Que su RMS es constante.
b) Que eL consumidor nunca aceptará sustituir
-- --, r- r--
Ld ll L I Udl-ltj¡
r- UIl I^l, ICrl
Uti i -- pU I .lan[
L idades del OUrO.
c) Que eI consumicior valorará más un bien en términos
del otro cuanto mayor cantrdad posea del mismo.
H de otro e el consumidor valorará más un bien en términos
cuanto menor cantidad posea del mismo.
e) Nínguna de las anteriores
\q
Marque la alternativa correcta. Considere el caso de un
consumidor que destina toda su renta al consumo de
productos, a y c. Dicho consumidor adquiere actualmente una
cantidad tal de los dos bienes que en el óptimo se cumple
que 1a utilidad marginal que le reporta 1a última unidad
consumida del bien c es el doble de Ia utilidad marginal del
bien a. El consumidor estara en equilibrio si : (1 p)
trl nra¡i¡ ,l¡ :- *l--J
a) L l- l u - , lE - u5 :ó tL.,]. Ld.Ll rl¿l
-lur nracin ru
P-LCL
Aa
L.lc I .

b) precro re ? es el doole del precio de c.


\ ienen el m. SinO orer:i o-
fte) tri prec i o re c és e-. Joble del precio de á
Ninguna de Ias ar---er: res.
-

t-0 La incidericia de un rnpqsslc, recae más en aquellos que


tienen la curve más WÁr.W--- a dicho f orma, e r
aquellos que tienen la curva menoji -d,e-ótra
..ilÁ*.i.ú...... De otro lado,
la incidencia de un subsidio il¡{it iu *uno= , aquerlos que
\\ tienen -a curva nas--Wl.(-.... y Qenericia más a aquerios
que rienen Ia curva más ,t)vUhluw.r/ (1 p)
1r. DESARROLLE Y/O GRATTQUE, TNTERPRETE DE SER EL CASO, Y/O
CONSIGNE SU RESPUESTA A}ITE LAS SIGUIENTES SITUACIONES:

1L. La f unclón de denanda de un bien es la srg,.liente:


Qx = 10 (5 - Px) / (5 + Px) A' lespecto caicule y deter-nlne (se
sug iere que utilrce derivación para caLcular AQ ¡,./ LP,) :
a) La e,asr-icidad orecio de la demanda ie este clen.
ffhq b) Los valoi:es e la elast,ciiai-precic cualiio
t:recics del blen son .i-o 3 y 5 unici. irLcnetarias .
(1 p)
(1
:cs
p)
I

1-2. Los gustos de un consumidor se conocen por Ia función


de utilidad siguiente: U = (2X + L4) (5Y + 1.s¡ttz
siendo su Renta Monetaria (R) de 224 unidades monetarias, el
precio del bien X es de 4 unidades monetarias y eI precio
del bien y, de 6 unidades monetarias. Al respecto calcule
(se sugiere que utilice derivación) y determine:
a) La situación de equilibrio de este consumidor. (1 p)
b) Si eI precio del bien y se modifica pasando a ser B
unidades monetarias, ¿calcule y determine 1a nueva
situación de equilibrlo'? (1 p)
c) El Efecto Sustitución (ES), Efecto Renta (En¡ y
Efecto Tota-¿ (ET) . (1 p)
d) En función aI Efecto Tota-Z (ET) y del Efecto Renta
(ER), realice eI análisis respecto de los bienes x e y
tal como se reaLízó en clase. (1 p)
e) En un solo grafico muestre las situaciones a), b) y
c) (2 p)
13. Suponga que las preferencias de un consumidor con
respecto a dos bienes, X e Y, se representan por curvas de
indiferencia convexas al origen rbien X en eI eje de Ias
abscisas y el bien Y en el eje de las ordenadas) Un
individuo está indiferente entre Ias combinaciones (X, Y)
(3,11) y (X,Y) (10,4), y en la primera combinación su
Relación Marginal de Sustitución entre Y y X es rgual a 3
(RMS'x
-" "^T : 3) . El individuo PODRIA estar indiferente entre las
dos combinaciones n,"r,.ion-ffi lu combinación: (2 p)
il (x,Y) (5,9).
b)
c)
d)
Debe desarrollar y marcar Ia respuesta correcta. Para ello
debe vafery Oe curva de indiferencra, ubicar
Ios puntos y la curva de indiferencia y además, consignar
textualmente los arqumentos o criterios que 1o Ilevan a
estabf ece-r ,su respuesta.
-lí. I
Ul\\iiVEItS ID AD DE PI U i{A
/-V.'
FACULTAD DE ING]iNIERI.¡!
u
///-
ECONO§,IÍA
"/'
Examen Farciai l'lombre: rl{¿,¿,,'frcr }t' QC,tt¡tr"/c. I¡,,.,
i. r,:i I de Octi-rbre de 2Al2
:i.rra: 8.00 a.i-n.

Se recr:ercia que las respucsias alltjsiz J//c cun corr3cior c'le ri¡lia no est¡tn sLr-jttas a reclantc.

Ii.esportda 1as sigLrientes pregunte-,;. Coi,tqLtt ..,I r.csl-rLrc r,t eit 1os esprcio-s i:lr l:l:rncr:,

1. (6 pLrntos) Lls curvas cle olcrt¡: \'ilerirr,rrrda cjrl nrelc¡.,clo cir pescaclo ilescr; \,re ¡e ¡
dacias por-:
Q,:20 000 F
Q¿
: r20 000 - 2úr 000 P

Donde P es eJ precio en Soli:s por ).iio. Q, i, Q¡ i:r ad of'ertaCa v demancla


lespeciiv:unentc expresad;L err liiios a1 lr,tr-
a) lln esle rlercaclo, e) i;rccio/.1c u.trrilii¡rio es ia canticlrrcl rJe eclr-ri)ibrio
e q=Qg_?(:CI tts) w'a d.aúg;z

b) Ijl ingreso total del aparlaclo a) e,s r_f {x;_'¡_o__!ríe:


c) Si sc pLrblíca en ei diario de nrar,r-rl cil'cLilacjón Lrn an',ri-rcio cientíllco doncle sc:
descui-¡re Q)e/el pescado coni;iene r.isL0s rle elelrc;lt,ysaéxJcos p;tra el ser hullano v
ia-nLret,a,lt¡r-i,a rje dem¿incia es 30,0'10 - lC.it00 l),4:l i:lrsvo pr.:ci,-''Cc cqLriJjL,r'ic e-s
*!_,t'j'a nllevir canticlacl tle ecluilibria c,s 1!.9§ü§ ,,.,,''
i) L.i excedente del cons,"lrniclor iicl apnr-kfio c) trs jgrpo.cl'_ - ir"l t rccclcnlc di:l
l-.r'ociuctor"del apartarlc c) es !!,gg_il ,¡1.

r i,,1,r
;Ü ""
2. (zl pLlnlOS). L¿l CLirva Cie COStt., tOtal rl .rrn,1,-r.c'-, rr,rrl,tC,trtia ilr:l bir',¡ ¡ ¡¡5;
"
I
cl'=- 4Q'- 4Q'-i00. Cr = ñz ;C i: ;
L¡ ¿t- Si la cmpreso r-s cornp.t.:,iy,i;).óLre el pr-rcir-, cle x que 1e bnnil¡ ir,:n:ficir-,s
l c;.It,',nricos ltLtlus: t I_ f_{., . ...._t
b. Si el 1;recio de x cn el rlcr'yclo cs JtTtf'. ( iri,,rllt' ti,s belteflcios cic,irslr compañía
cornpetitiva: \b ,' 2-C

rí-- 3. (2.5 pLrntos). Eu el merc¿rdo dc auto:; l¿,i cLrn,¿ iic demanda es f,:ij -Q ), la curva cie
[-'-,r'/ ofe,ra es ¿e p : ]+ Q. Donde i) ,.rS e ' prccio ¡ () lli ctt:titl:t),/
' ,/ a. ¿CLrantos autcs se tra:rzarán a rr ')r'eclo ri¡ 7'' \ V
..,. . /.f
¡p\
- ^\ b. Craflclr-re a detalle yI L-\P',r\¡(rr
expliqLre l)ei Ji)arr
aparrado
rr:rrv a):
:l/,
frfo,D ,) d^ Q-i"Á[r:hro §i( l'tü
/ ¡ L, ;r\ttb c$¿' e*¡'tLLih'i' '1 .y ":uutd.r,.c'L d'* L f'^n a v d-tt'
dL WilLo\,t ¿§-t. ecuctct'o",ie's i.'t u->rva,-
//'
wcrnrdy' ¡ o y :ki ,rÍ!o?,,ft,# ,u "tu {t:i-, (
(,4 --- \f**y7
u- -'J- "*fffT;
,
podtnt-os
";,rffi ^'-yl=*#,Wyffi,,;
brP$,1-----;-\
l

', ' ./
./ ,
6;ar,l,\*tctar{
ffffifuf, H;ú ""'*{it
'cüt !4^Cr/^ dq
úd CLL}*TA/t't
zá^^44¿a^ot ot"44
' ,r/;\; de WY * LA yara 4<- '2^t"V*§cl,
(o Poro ?' ?1 '.ot
=tr,^
ór, P*L,.5 \*uáu,+,..
,.A- l-f,d-1, l" j
l,l
iiri i
L].
:.i,

4. (3 puntos). Crafiqi.rc a,lt:tirllc 1,e>;pliquc las cort'sccr-rencias en el iner:cá¿o ¿"i


l rii iili
,il;:l ii
rilii
ii
cigar-r'iiios Cc:
tlltr
rl
a) LJn ar-lmento e:i cl rrLccir.r dc1 gas para encendedcr"es. i
Iil I
I r:.1
I
I
t ,i

F rblaÁ't¿
I
A*^*., t'*, d,et fi-,.car) dz.{ 6os '- Pl?r-'/ot't Qé"
*Et*ctarl.d-oos.<-t fft {rr?o e*F!";*d-o''-,1 1.
/'l L.ps o;.qa,¡^tLtp¡ Eo7^ E<e-"s W
( . /-') ñt
t'
t a ÁÉF,óN0A
OEfPLá"4 Es dco| o@ ln?*'e" lr+r §.< *l- kwa ¿w
,"'./" f+
/
^ Ía ^
A+¿q,\ {, "Á;* e r4ú t La d<;ñ^aa' "us1'1114r'
targ 0447 P
( irr\sitqrYvúru.§
^1e.,i.^1t.¡:-rt
\
/
E-' i¿a^,hrc{^ *¡cldrtic-
c'"* U lt^el@dt d'r c-fiO-s;Ü
,1,r"*. 4 ,¿»¡.p VaL-t fu fra?b e
g(- Lo'l¿-(fiarq' .ryu
L/\ J C,v
c)i- t^tlir',
(Y 'rü--* ve-rd( a$'ck'b d¿ *u^'
á¿r*wa^4e
T;
?

\/t

r*
5" (3 puntos)..En el ireicaclo Ce ¿rlil¡entos iiay 2 consumjdores A y B. Sr"rs gustos
Án,."r'ii.¡'. l,'^ ci-,,i,".i.' -,,1-\r..
\!tJ
.1.,:-',..'".¡..
rJL \¡!'llottud.

P:50-2qu
P .= 50 .. clu

,?, i)o¡ldcp es el lrrecio dc cquilibiio,r,q., g¡ son la-s cantidades demandadas del


iilciii'icluo A ), i¡ l'c:si:ectiyatnc;¡re, si esto:i So¡ los dos úlljcCrs compt"adores:
I-Iallar el eqr-rilibno cle liicrcacjc. si ia c,.rlí cle oferia dei nrercadoviene daCapor:
P-ls,ei _,,/ --
Prccio c1,: r:quiiii,::.,: bÁ --" ,''
Cantici:rcl Cc Eql:iliir; ;u. _*=_2j__/_

6. (i.5 pLrntos) CraÍ;c1,;e cJ cqr,riliirr-io el el caso cle una erxpresa tlonopólica. IndiqLre
clara;lrei¡ie : u'e cjo dc cqir;iibno. cantidad 11e equilibrio, pérdida cle eficiencia:social,
excedetlLc dcl cr¡nsurnid,.rr 1, c:lc.eilentc clcl productor.

ú,rapo¡¿q=¡¿^rL.-út^l* - wv\ cé W e.) u<*',d¿,d,n< d'^ ,;, bu.u ov<-rvrr-+.b


i,t//
ltt, ,
;;. d- , ;*frr'rc*"
nnr¡r<,o,d"o,' ,\ fr"'rrd" §/( r*"W {^;e.ñ ,^a.i;.*, ta
*.r;e.^ lc ot^r corn¡,t^ t^^dfrr
o,[ .O^¡,fnnrÁd,af
ar.,,.r ,,1
*Y'. "n-*
A**y'an't, o^ y/§,A,§,/\ - fo'
)at^^i.^hr.b t h
^ pa/&o. oJco,ruz-', e^ l-ayALLÑ ^
i (.,r* aar*"*U¿ eL* lx<rv,r i,,:6', C t{e(l :
{vgr¡¿ácul¡-
C r\n,,{ d
"/.
,t"''

n9*
t"
N {r*l*M
o\4 ü *\a«/J'z/"o

& ,"' ? e{rl'col d¿:


7

L ¡tiv'ersirlud de Í>iut,u
,.
Fuc ultud rle I ngertierírt u),".
i r¡
{}.".

t i.. .í
Curso: nCOxÓuÍe (ECo) I -, j i{P
*
!/., ----
Tercera Práctica Calificada Lt'
l_r
Fecha: Vrerrles I de noviembre der2o13

Apellidos y Nombres: .,'),it,.!.:s.. i),.,.¿.,'-i,.-r{...!..r.,r.t,...,ifr,j.i, ..-:i....


La presente práctica tiene una duración de t hora y 40 minutos.
El puntaje de cada pregunta se encuentra especificado en cada una
de eI1as.
No puede tener apuntes de cIase, fórmu].as, ni otros materiales
sobre eI tema. sE AcEprA EL uso DE cArcur,ADoRA.

r. RE.SPONDA, CONSIGNE O DE SER g4so, su RESPTjESTA ANTE


I,AS SIG.UIENTE.S PREGTINTAS :
-;
i" \,
'¿
/" \r
1
ES ......:.n r !.vi,.d1.....
1os

2. _r r:slfis!_q, lllá - -f v= - oló:T,L r.o


siguienl_cs +TrT)+

\¡ .-tli / I I \
..,-.u..r.,...,..i..,:.;..t...
^-

3 las qlle un goble::no puecle


fiscal y cuáles de poJ-itica
t,
,(1- punto)
.:::.: . :!,:: . .{: i.i:.. "

{\
fL
I:J


j"r{ot .4¿ [4 J iqtuin'¡¡"
" " ""'; " "" " "if "" "" " "";"
6 ..
" ' -'u'Y,','-í.v-1
t.!..1 "..,.. .4 t.. /t !.*..¡.1-<2¡, ¿. .........
1 c. (.
Jtl "
ff&artb.Í¿fos6r
PrL
úno .
4- Considerando su peimanente lectura sobre l-a actualidad
l^rr.,, i^ i l:'.i 1
lP uú!¡l¿áJ qir'. lYly'- r .
l

.lr,,t, l , i*I¡ ,
económica nacional y atención en clases, ¿cuánto es la tasa
cie crecimiento esperacio det Producto Bruto Interno ('PtsI) y
, d_e la .,inf lación para el pi:esente año " Qué puede.r comentar
"t sobre las causas de su variación del año. a lo largg

':.:t..:L..{:.1

\$',

5. COlfrpOSr Crór:
cle lcs bienes que conf c i-nal: l-a canasta básica áC-u,.-l11 cie in
oLré
*'-r - --**A:,Los f é merece?
comentar (1- punto)
ñ,
P)¿ tlt.tal ; -i
-,1 ,, . :-.\A ^"v- -1.i,¡¡ 1a '','- )
';/'-:"¿,:.),¿t
hci-, 'it t-:,!¿--
" "" "; "; " " " " " " "':/
!¿i::::'-::......

ó

-r
6 . El PBI nominal vaiori za ia prcclt-lcción a pr':cios ""'':'""':t-'::'Í'(!'-:1"
nace a prec
ic hace
Real Lc r ()s ""'-":""''""^""""""
pleCios
ni en:r:as que e L PBI Reaf ......4.;....'.,...'.'!':.':.::::.: \

ir.sllritsinc, eL Deflactor de1 'PBI es ei itlCrce qrje micle ei


ó rrlr,,e I . gen3ral :.. Cie precic oe
L.N.
'i\ IPC, es el j nc.irce q'r= rL'-Ci¡- el-
Ce Precios aI Ctnsumid.or
- ..r --^ r .-r^ ^-^^ i ^a ¡.ta ':14 tí'j''.> , )*-'^--.-.....)-í
t/1" §\¡l*r+,*no ,il l*"-
7. I.a cont.abilidad nacional se basa en 1a idea de que la
cantidad de actir,,icracl económica que Se realiza en /-)\ un
r \l periodo oe tiemp,o puecle nedirse por meclio oe Ires \Ji
LN métodos, los--/í"t", generan, medidas idénticas de 1d
cantidad de a{tiv:rd.aC. ecoqómrlra- EIIos son: El máEo,.do deI
tr ..-...........f.*:vk,..ix/, de la ....t',:':,*.ryid.-..-.)...... y der e.{,';Y'í!}?'M

B. La drf er:encia EI t r P NB y el PB.t radj,ca PrinclPalmente


en -lLle e I Pr i llLrre: o 1t- r,,alora ios bienes y servrci os
finales en u.in ria eQ un periodo determinado
cons ide rando 1¿a Ce los generadores dei
lnrt
proCucto, miten inclicador lo valora
ñ cons iderando I¿a v:.!.¡.r'¿j:::....,.. f ísica en el interror de un
pals de Los g ern( r:e s del- producto. ( 1 punto)

9. En un pais Puede ev.is---r pTeno erry7eo r^r1


u\-r u
aqt a¡ f .^t

signi fica ausencia L o,_ d I re ..t- sernp J er. . Ei desemPLeo


proCLrcciÓt en la srtuacrón rnrciai )r cies'¡ués de ra
api I cación cle amba s nLedi cla s de i - -a ¡Ls-,,
I
(aumento c,l e t y de G) . (l- punto)
t exrsl---rrt-e en per'odcs cie pTeno enpTeo se I e ccnoce comc
4\ iaSa Oe ........,.-'=:..'..,'.r.i.:,.:..1.:.,t.....'....-......-.......,.............., qi-e eS COí''SeCUeníl ,l,a Oe
una coiribLnac i aii Qe- deserryTeo friccionaT \. desenq>7eo
,Pbr su paite, e i deserrg>7eo cíc7íco .aesr-,lt-a
-
f-..,, ..........-.,,...,:.,.r.:.i:.:...'..'.:...:........,.
\JI Ce ia iife::er:-,a eni,re 'a tasa efectiva de deserrpleo li ia
tasa de deserry>7eo ...1.,....,...:.....:.. (1 punto)
II. DESARROLLE Y/O GRAFIQUE, INTERPRETE DE SER EL CASO, Y/O
CONSIGNE SU RESPUESTA ANTE I,AS SIGUIENTES SITUACIONES:

t 10. En una econon.i a en La que la Propensión Marg'inal a


-'! ,/ Consumir (c) es oe 0.75, ia tasa impositiva ,3ei Inp-res-.c a
.- i ' r -la Renra es del 20%, ei'l ni-,'el de ia Demanda Autónoma
i.

o
i : Gasto Autónomo de L75, as Exportaciones (x) ie 35 'i ' as
: Importaciones (M) de 29. Si ei Presupuesto Púb1ico (SP) er a
de L5 " Cons:-cieranCo que L a Jenr.anda Autónom.a, Ercortaciones /
Imporr-acrones y el Pr:esupuesto Púbiico están ciados en r¡.iles
g|e_@. Se scl¡cita cieterminar:
a) tri- lrygr!! en ia pi:oducción Ce equ i 1i br j o cie tin
:.incI-Cmentc de La tasa impositiva (t) de-l rmp:estc. a .r-a
Rr-nr ; 15 b ;-' Ln: - s tf I ce :r-uaIes. ln--e:p::'-+ i- en e_--;
ssrr§_iqr¡e lql__eq_.La secuencil anaiiiica cie .llSg-q:g-
g i ?_q!-q . (2 puntos)
b) En cianto l'ar:ia (rnCicue sr rneior.a o empecl:a) la
F.ecaudación de Impuestos (T) cle raane_ra cie-s-agreEacia l'
i'.rcgr: ia yqi:i-q!-1cl,r_"-jfsj2.l_lCgj:ll (in':i:-que sr :r.e j or:a '-:
empec::a), como produc[o cie ]a meiiia cie p:lit- ;a
f i sca'l antes incricada . Tniei:prel-e (L punto) .

c) Ei impacto en e1 Presupuesto Público (SP) cle Ia nedicia


cle politica f iscal antes indicada. _In!:s!rgte-l=en
q!f a _§gnsigne cuÁl_ es la secuencia_e!3fit1_c_L !E-ig
qe!§-:§-19!-!e" (1 punt'o)
d) Er-r un §g!.___g muestre el equilibrio d-' la
pi oduccróri en'l la struación rnicial y después de -1a
reducción de a tasa impositiva (medida de política
fiscal) (l- punto)
L1. Manteniencio los rr.asmos datos v condiciones de 1a
sjtuaclon inicral del ningun
dato cie los apartados). Se solicita delerminar:
a) El impacto en la proclucción de equilibrio luego de un
r¡Llrsrne$e de la tasa impositiva (t) del Impuesto a Ia
Renta de 5 puntos porcentuales y de un aumento del
Gasto de Gobierno autónomo (G) en 15" Interprete y en
ella consigne cuá1 es la secuencia analitica cle la
caus_a-ef ecto. (2 puntos)
b) El impacto consolidaclo en el Presupuesto Púb1ico (SP)
de ambas medidas de politica fiscaL. I!!§I!¡e!e_jl en
^ r r ^^-- ^ i
erra^ corsrq,le cuar es la secuencia g@
causa-efecto. (2 Puntos)
c) En gL__§olq__gféf ice Inuesti e eI equilibrio de la
I
^L ,n
Universidad rle Piuru
Ij'li ,'¿1
''.['
Foculforl de Ingenierkt

Curso:. ECONóMÍA (ECo)


Tprnar= I)rÁ¡*.i.;- Calificada
Fecha: Viernes B de noviembre de 201-3

Apellidos y Nombres : . . I4odq. .Qq.ü.,/.i. 1o .'!+.hc,a


La presenté práctica tiene una duración de t hora y 40 minutos.
EI puntaje de cada pregr:nta se encuentra especific"ao en cada una
de el-Ias. i'
No puede tener apuntes de clase, fórm,ras, ni otros materiales
sobre er Éema: sE ACEPTA- EL uso DE cAtcurJ\DoRA-

RESPONDA, CONSTGIIE O
SIGUTENTES PREGTINTAS ;

l)
\J

r-i.' i \

tl\.

2- Tnterprete, 1a siguiente identiciad macroeconómica de l-as


. :iErient.es cuentaé: (S' - i¡¡ : (c + Tr T) + (x M)

.,{,0,..U¿I.M*&...AnMÁÁ, tA *¡¡¡J (i[ ojl,o'r-nii {Yv,,¡v,Áb -9-a ,^^#Áun'")


1. l\
i. \,1\rJ
I\

3. ¿Cuá1es son las 'variables con las que


emprender accj.ones de poJ-itica fiscal. y
monetaria? (i- punto)
\i\J

\,1 '\

4 - Considerando su permanente lectura sobre Ia actualidad


1
económica naciona]-y atención en clases, ¿cuánto es Ia
tasa
3: ::"i:1i:::'^."'^t":';:. ;"1 producro e;u.o rnLerno (pBr) y
sofre i;:--.!';7e{.:: "":';::i::;.;.:
oué
r:;';3" ..""r-;;;3i
t¡:Hmá 4X»rl¿ (2 punros)
fl /"""""""""'¡""" "/\"""""'
,'av
i.''r,\ . ti0_d1'.¡0 Ewp;711t<rn., \á;1.
_ .,.
. -- - , .

i_
\f

5. Asimismo, Iuego de haber i--¡o.r- j^=rr^


-^r-,^^
:' : l
"¿
d e 1 o s n i.
"' J"-' {, J
"#J; .;t-11'. J'i-'
.='J.-..1 -'
-;. -:
::: ?
.
"'?i',-,,i;-:l
^
^1

.J;;t (1 punto)
..\i

6. Ei PBf nominal val_o::za. l_a cr:d..--:A: : ñyr^r^_


mtentras
=ient.ras que e_ pBf
pBf Rea-l- -: -
=^=
Asimisno, ef DefJ.actor del ;;, .=-
nivel
r1!vE:-L general
cfñnora . gue mide e-l-
n I yv¡_ela¿ I /
.-.J§.b.rLr^{á..y..!.(/,{um....fl¡.\o...!.f»noniru,,.,
Oe ,., 'de
. =

(1 punto)
7' L4 cgntalr¡r.idlgi
¡1
lracional- se basa en 1a idea 'de que ra
cantidad de acli,vidad q-".
l,')"* período cte t¡erpf\á puede""orrO*i..-
medirse._,p;; "a realiza
*.ain
-;;-Ir"" en un
métodos. los\y'irhr; ql.,".rrr, ¡rediOas idéntjcas cle (3Ia )

ca¡rtidad de ag'É:-rtiaaa
......-...?hT \l -Já-il- elonónic*,í er-rol ..,..
.)l E1 mátodo dprr,-
*xr._i.t1.:....:';":.r voror, ñr, nr'.,rt r¡t_..
"or,,
(1 punto)
t
* gue
en et primero de er,os,yvalora
3,15":i"::?-:::":^"1,:I: e.r pBr- radica principalmenre
*"I-I;¡iJn5"';-;Iffl':l;:
final-es en una ecqhomia ; . -,; *' 'osu,*-"
_\ i
I-'i\,'
cvy.¡v4uv!qrru\r rl' I....rI^{.rFJ\Kyl.\J*.i.l\r-q)O....1......
o n s i d e r a:r d o rc. .ftn.]g[.0\s.Ái*r-:?., de
^t 1os 1"^ - 1: l..*i " 1 d g
generadores
producto, mientras :.&ue\ 1 de1
-;;:';i; ;;";á
,.
qorrr:nr.rn^ indicaáor
"l segundo ^r..j ^^¡- 1o valora
.i
?l -
: ?; ;1.,
-^

"i;
::l:'::'il1'-
país de los generado 1: ::tSñr{1*S delproducto " (lpunto)

9. En un país puede existir pJ.eno anF,)_eo pel:o esto no


significa ausencia total de desempleo" El desempleo
2
-!

existente . en periodos cle pj:qno.r/egg>J-eo se Ie conoce como


tasa de 9\UAnU*f*f. '.,..1fi(l*fu-t1tr1.....L,Í-....,
gue es consecuencia de
una combina ciórt' del deserrytTeo friccional y desempTeo
......,94,W1+-*iltfi*.¡,,:.'. Por su partJ , el,.,á.."engsleo .í"1¡"o resufta
de }a diferencia entre Ia tasa ;éfectiwa de deserazpleo y Ia
tasa de.desry>t-eo ..-....VJ:*.tUtCL0..!¡.Í.... (1 punto)

tl .nnSerUrOLLn y/O e¡UrrlQCrr, ¡¡irnnpmfq Og Sgn rL CISO, v/O


CONSTGNE SU R-ESPUESTA ANTE I,AS SIGUIENTES SITUACTONES:

10. En una economia ei't la que Ia Propensión Margrinal a


consumir (c) es de 0.?5, Ia lasa impositiva de.l- rmpuesto a
f) La Renta es del 2OZ, eI nivel de la Demanda Autónoma o
Jt
1/
Autónomo de 775, las E¡rportacionés (X) de 35 y las
-lt ''1 Gasto
lmportaeiones (M) de 29. Si eI Presupuesto Púl¡U-co (sP) era
tr'I de:,1-5. Consiclé::ando gue Ia D,=manda. Autónoma; Exportaciones,
fmportaciones y e1 Presupuesto Públ-ico están dados en mil-es
de,.¡rillones de S/. 'Se solicita determinar:
?) El impacto en l-a producción de equilibrio de un
' incremento de la' Lasa impositiwa. (t) de1 fntpuesto a la
-l
-t l'
i
de5 eR el-1a
ne cu d I caLlsa-
e a 't
C
(2 puntos)
=:=c=_c. varia (indique si'inejora o empeora) .Ia
b) En cuanto
Recaudación de Impuestos (T) de manera desagregacla y
Ir-iego 1a variación consoligada (indique sl mej ora o
empeora), produito de -l-a medida cle politica
fiscal antes indicada. fnterprete..
-iomo (1 punto)
c) El impactó én eI Presupuesto Púb1ico (sP) de la medida
L¡E IJ(JIJ LI(-O !f D9qa Interprete y en
de politica f isca.l- antes indi-cada - 'analitica
q¡rLeu f rfu¿v *^'"-*r-::-_L----

ella consigne cuá1 es la secuencia de Ia


causa-efecto. (1-.punüo)
d) En un solo qráfico mue§tre el eguilj-l:rio de -l-a
producciór, en l-a situación iniclal y. después de la
ieduccj-ón de Ia tasa impositiva (medida de politica
fiscal ) (1" punto)

L1. Manteniendo los mismos datos y condiciones de ' Ia


situacj-ón inicial aeI caso áñlerior (no considere ningún
dato de los apartados). Se solicita determinar:
a) EI impacto en la producción de equilibrio luego de t1n
in.ién*nto d. 1a tasa impositiva (t) del Impuesto a 1a
nenta oe s puntos porcentuales y de un aumento del
Gasto de Gobierno autónorao (G) en 15. rnterprete y en
e1Ia consigne cuáI es 1a secuencia anal-itica 'de Ia
causa-efecto. (2 puntos)
¡) nf igp_e-g!-g consolidado en eI P¡esupuesto Público (SP)
de ambas medidas de política fiscal-. Int-e"rPrete. y 9n
erra conriqne cuar es la secuencia enalitica de la
causa-efecto (2 puntos)
c) En un solo gráfico muestre el eguilibrio de la
3
producción en la sicuación inici.at y
después de Ia,
aplicación de ambas medidas ,cle poJ-Ítica
.

(aumento de t y de G). fi sca I


(L punto)
v

LITUI\./EtrISItrI^Atr' T3E t3TLItrT^A

ASIGNATUBA
THABAJ, ,. 03-
rr A^ C^,^¿ FECHA ,l b ,/
N.MBHE ALa ]s a) "l 6, ?
/
lcr
I l)2
, t/ FIRMA DEL PROFESOR O AYUDANTE
i?
CALIFICACION
?n
RECTIFICACION FIRMA DEL PROFESOR

o Xo = l(D.
Yxo [0
o
rxl r) Yl= I1o.

f
U^1"":o o-\r, o- + t5
^ [ff; = Zá 5o
:---- | ? *o* V", 1

t a- J \ 2-/
T- Julc-*tla7--ü"=
\¡, 3,.,L v1?.¡Jtro porryw ='--Lq,r.'wo, ^,¡ lu c""t'ú
Li D¡c-v> .t¡'lr.r-" ,[n: *\r,.r^'^.,rc^.c,t¿.., ']-t 'vl¿^^^r-{o\ ,inu-e^zs^-
--)r : -/
!"--*,.* J uc"L-r r't=oLr"Jo /^ i'^ at

nl¿'t-'
i no[si5f1c,O. \

G^ p,t, cq,aD ".\ ?z*^irvr.acrruc 5í V cr^ayr*fi ) l** , qJ crz; w- *.,\


vwr' D'¡* in[r+'oa
€,-,Lr*-rÁo {-,-gf- L, vc^ ,' C-c,n-'o q
@
foJ^ J, .,|. á'-"^ ',^¿[ís1rrc vJgsó1"os =.L o^^'ós ,f aL i1"J:
r¡lrt-r,r

y^)o. lnffo vc, Jo.1 an'J


1, ,r: f:r:
I t (, ,, ,,^ , Í*rt-d*r,
I )" ' :t
, ',IT --1-
M _^o^^-fo
_r

I, ^ Do^^y Jw-lV., V:y» J


l"*cr¡os fa*o', "JY"r svn-;Au, [* , f

nu'r-í
ftu1ar r"naf,rr s.,ttcl-a ,L. ¡*L l-o c*¡,,bJ u{r,-to"-^o/a''/'L
Or

Ll
L\ oiio'-i .arb
\-r <;t". C
J ¡<*o
¡**o (,u--t
Logrc,c\o-'
\o
d-t s

.*:
lLt"'ltwJ\L)\Lr'\u'\
ua\-\
\.cr v¿[cnüo^.,t t1''t- ¿>isQ' 'lÜl^rr{ '{t\
\ LL/Yvul!L'\---

\./Acf.T-¿u*Ct 't/\
r \ )rt- u"\c\ 'rtl + Dv¿t'A4 \
\ C\-r-wtr-n"-'cY\C'c\a i,cn "<a*c9"-dl f}a9At*t+ I
'rlum4 ', t
l^ L-r
\ri,?r\ ) /"/' ' I
CP.-;] cl ¡f-"-C.- ¡'(c'l' ¿'t A'- )^^^bl
p'o¡u,^^tuc,.J c'L' \c,.
,(CLlv Lcu''A ,t.l tc"^-'i¿'¿ 'c'Lcn
-"-"'-'
J""-"1.0 o t{'' 1o wr-lc^- .
Ponct'^^-lut-l

A-i" x
o\

Q Á'ta
G* p,ry=

\-or\o,iir:
\ LUJ J\ I'- Y:)yt
) a---.,^!-^\-/,\-'
b; bo1.s. 'li:ü
"&'"*,'r1:1i:l^:.11
do^' J e ze&
vt*Jr¡
\
,\t UvrCr- [/r€Q,vtcn
U)
A:;;[-du lD\

) P*o. l-op tAá


/i,,,1 *r
(,^* o}t-c,," cfus 6t,L= cla sitl q¡^t'i oa'"*a
lr-
o tluuils v10 r^ncr"- +'1 P>)
, L-' t1, vlorw'.cr-Ls o ¿ "\P <]

tr¡ {u
a\-/+
t__--,/v--0L-

;* "',lTy:': f
";: r*y::Jp***o-
oY,1T
d.-."':'J
, .4ia"-J 'a
^('^- \ f
&^^1i,1-
ñ::il
%¿
;,""I^a-"^ ' ,1"1ii"
t ^Y: :-1,,:: '
r¡|rr-,cJa { v ,n-nuS**'l
á¡*-"''o-
inqyuf2 tk
!_)
r),-tc,t

f n l't; o-""" ' ,


tl,lr, p( i.*

It , J no
+b1t i"|z;¡"" ' fi I
f.a r,bs Ied*r\ q ,l cc.^, 1-¡*J "i
- oe I.Ju'rni'rir-i ¡P'crt*l.cl;1f-z^l'1;ü"\t*-
^--l
,
p r-o c'La'r cr
QYoo ^ ' v] t1a1rv ..r'. lluri¿ *t c!'tc¡ '
\ 'f'* l

/Fl" .\>-) L')o"1os ,Po'/"clrtic'lL¿a I-^..o


\'lt
a/ \ o,
I
I.lb
rU,
I _),\ ,

üsc(act \.a
5o ü¿c(act
Lr^
\.A ggr
f X- c,Lrl..-P
c'1,1.-p
i

,.,. 0r'*
rq&Lr'261.
nfn"qo,.L,' 0L'* ¿¡zltnqc,.L,cl,oc,t
f^ -o' \
¿¡zlt nq c,.L,.cl,oc,t ,,f^-o'
ctv.-'..*","¡) ¡Lis,,o.,is ["'.
tt rLi5,^^0.,-10 o¡fo. ?or
[". off,.fo. ?n, l-,
l-r'l'-
1o'-{-
- '\\fffi"r-l \rttr
,*-\,,qÉXI i \ J.[r,.,- '" r], ü c-."+ iolud
ul? I'c. iolucl p.outuu d^
ata- I
^svn-á
," c-.^^+
ry]=" |
rko|llll*.o".*) . \ I r ol
o t,L,
cL:,,..,," >r(g)o#{6 ¡r,*
^^,,. ", 2!;#íit,d
u;
¡p*
lt",,^fr(,
0,n,,^1,(,
)

..,1:*'.,
afl.ftrru ] l,. tr,
tt-vu {,1 pr(cr,)
,k* r.'la ,, -ln
rrc.l,) fr'!=lr,
fr-b=lo
ffrrc.l,).T-, Jn ,Lru
,(,ru ,Ü^¡l"f
-tn ,Uru ,Ü"¡laf"'
,ün¡b,fr'. ",
¡,.,ocb)
fh 6 cü
cr.,,v
: l:',a
o[¡..*u frc'u
P[F'tq o
f..,.u'o I,e^ z*, rt'"-*'i
i z*'rt'"-*'i l'^
y Al e,l,a, l*n" h, )c, *"t'.yaL)yrl
J

ve a o.
\ I,JT,,:
f

l¿*.-^.t*
t
'C" ,+u
¡>rcc4ruv \
/
I
.{.,,.,.1,
,",..r1\ =o.,'^il -

L )',.7 -:,,-7ia, ), C..Jr,a ,)J Dlf J^ Lvn".rn Ju \¿q9\lr6lc" ¡ pxrr^1o, ."í'4-,!& A ."*
' :.-,-
' \
ü,-Y =-., - ,Li\^-- tñ..r-
-)] lL-^
I L Jr, a"*"fo rO--r1' ,.,
(l ) - I / /
ti fl r , ---/ \
"', / I ,un' .u .t¿4 a i;rl
,) g C'J{ll,-ci< v\ un
v/\ p,
p,,.üo l't Dtrr fr*hx.,
v\'''?rtcl
^(rl,r,-,o
()^
u^ ;*
"""':-' ^
fn-"
u1
Jt:ti
**f: ¡rro \pcc^Y
=ouJ \
, two( YL \,) al q''"1'
&\o^. óAr^.--o i-, !n¡s.r* ,./-^ src' e c{c-at \
, j

i fin,_1c.üzq a*( ( q^e se'


o- L( yt* d^^d*' , l*t^fr,,'lol',1 "''a^) ''o
il ; ¡",
Ac,aac(uc/ '@,i=J-',*
fn'o ?u-ü\'* /^" c'r-+^' 'fr '{¡dos *u'': Yl
,ucJ,rrcl^,-, Srs a$,, ))', A^^ p"6 C*,t =r pv'eL:l,s- in{t"' "Llzo' /a gln'l&
c^^^ /o^ *q,,,1 o,
/\-YL-lv\,* cí,- .. i'-P ¡*)os oL, cp'¿cci i* o [*::^*t '/A I
t"-/
C.odl D!-,!-: v1 g-rü o.r
\--:-

Fp,EJJo= Vt^ys I* ^r"RlrrLú. v\;


¿'
lü ck,-¡c"-^a,lc.. Y-'1L¡Ci_
t

*'l*u,r;i yn- r¿-cl."rt/¿, -'l.' a'


r . t
r,\,\ Lq u{ U -ct cioí \ps- pqJ¿ "q,rl-; ' ?'t ¡o*"L'U i^-y(rü- LLt v:o
}/l C, ur\a a,r=L1,t-xo- tlq, d,a <?
f f.L"
l^"0'!,*
-
)*
I
Ar-^a-,..^.c(, 0- ac1^t LL,p
f
i 0u u{,, L' t auo; c,L
I .-Yvlvvl

ü0,,0J f-r,--
I

- -' ew\ ot t-¡'r Ür PPn 5 o^/\q .{,c,Lr,r 5u .,iu kzo co;


V)o.. pa-T- oe
c-0
,-" P

_l -----l
nlo
fr.idlyn,
cfrcrr
!¡l .§^/, Vty, 6e^,t r¡>os|á(a irn¡r,rti
ua9 -4.,..¡ lowjr Ll z'o ü0.i, \Dar pcnrL .1, ^Jó
' [00,rat.",,\cL '¡ I Av,lL;'a;,
fr?'+ürr Áiuv1,os o,^^ rl
!-+ ,0, u,*,)ll-l2.,.¿ oú
Jl
rU ob g]"cc't

c1,tftc,S l-.it-"1t,.t
I

Ls "¿,-.
xc
f:s:Lü irv.f.rrclir sur t)<.o ei^^'; ru, v¿n'.rLi¿*¿'(s-,h ule l, Ie
( &
ó.?-o[los L.i a"'*, ,u* l-as
r,^^l
lcn 1,-r
-.--,-¿ ^L Ji J'-r.''.tI
t'.t
.1 \ L¡,'t^.
UC, AAr
¿ -i--. p 0,
trY'
t,' L\
o/vfQ
v\ ,¡) Ll§pwr C^- t¡.c.tu,.c¿ -A.t'- t¡l , Li 7cl"roí Pr
, ! f -. c\l - /t .-.- I' ¡ri Q-c,-.s-r l'.
S!"rct L[_¡.^r/- go6
[o
I

I \n (cn ola c-.-.^r' L).nc, n*={-,\A-


t"/
n.\^ J \
/ !\,' u'cc\ /
't

l¿r3O5

awr C,c"",S;cLr nc,ctqs ,t.xCLr.it¿e,

+..c,; [.- pv
lttl,
eoL^ ÁaWW At-rt vl t^'V ? f a c uo--^cb unct ftre!
.0

)-tc"[.i Za Ur\cl )r
,¡r^**J\a.{ vl la.t s r¿u, /^ t L wlo...r

ol¡o alr.*l v
-
q1^-{ tiut, ) !"1 39^ 1t cl)-rc^cLo s tLxcl u tL>bn AJC S-2r-r a2^^<l C
=p/v,
, I . n

cLt^1&"^ Cq-\ t 0, üüv su


U.
\rvt1[oa .1'^-\ Á0- ]uQ..!o irw¡l?cLbv Use-
ll r, ¡rrs,lrÜ f*
fA oir^t" ?c\dol fu. 9n loir,.^-
P!.r S 0^ a. ¡t-\O ncl,*na /\^ Yl, [^z^d
d&r^^
?w P^tu ü S'^
IJI\¡IVEFISItrlAtrl DE PIUFIA
*
ASIGNATURA LI*N*,rrtI"O- TRABAJO ruO I.i
NoMBRE 9**¡o ,ffsnn* ,&*r*nu*rt FECHA iq Ío q I tq

FIRMA DEL PR
F O AYUDANTE
ffi
CAL¡FICACIO N RECTIFICACIO N FIRMA DEL PROFESOR

q)
gP,= (u-n la., : o,Q! pw A)

/v? /*^ !.0-

ñ
ffiv k 1o

- {.lc-¡l
' = ñilx
%* t6
(** o)
]0

Y ,y), t^n\ Jn;/"


r\v
o ?cu.*y¡ = - to
xE (t^ 'A)
lo b
9)
o
\.Cn3YX=-lO yL
lo to

\ tt' urL h,^


c) Qx

@t '*

)Qr
?
o b L*
¿lo

0
e) '{
q

b) f* ,h" ,,trl* ¡* qrf"fr4r", /n, /rrr^l**'* 't'# re l'"


\ ry li' 'de f¿us u¿.
W- ,01p\
e)Qx=a ?y=,1 y=
Uxj !k= I
&

t:
-)ff , r *
16
§,^ T=
ilñL ,,^ntdn*¿' d)t
N, ePúr.a» 1,*
dtl" ,lit,l" y I

d) ?f = ,r Qy= I y= rr"

Y i !k.b
4

): 19
I
= tL

/-Wn, dilfrb
drl "[fi¡/ Y 3
Y ttn;*d'aw d'l Jir^ Y,
:+: b
Universidud de Piura
F ac ultad de Inge.niería
curso: gcoNó!,fÍe (Eco)
Segunda Práctica Cal-ifieada
Fecha: Viernes 16 de setiembre de 2OL4
\qf!
ApelJ.idos y Nombres:
La presente práctica tiene una duración de t hora y 45 minutos'
ul puntaje de cada pregunta se encuentra especifieado en cad.a una
de eI]-as.
No puede tener apuntes de elase, fórmulas, ni otros materiales
SObrE EI IErrrA. SE ACEPTA EL USO DE CATCUI,ADORJA '

r. RESPONDA, CONSTGNE O DE SER EL CASO SU RESPUESTA AI{SE


I,AS STGUIENTES PREGUNTAS :

Cuántos más bienes susLitutos tenga un bien, ¿qué Ie


ocurrirá a la e 1a st i cidad-Preclo de la demanda del bien(-) ?I U
L,/E L

lado, ipor qué'-ée dice que a los consumidores les interesa


fos productos que adquieren en e1 mercaoo tengan
({\
,
N\

2- Por la e-Zas tlcidad- ta 1?= bi ser: a)


e b) En el primer
caso, el valoi
si el
CaSO, Sa vaIOf Cte la
de Ia elasticidad-rent
elds LrurL-rdL¿--L Ürr qo encuentra
enL re O y 1-, Los bj enes serán ; ..,-l*...-.*!'l'''7""1u''i' y si el
,,r,(...........
valor es mayor que L, entonces los b renes serán
-tt{
..... v.d"..... 114,14&..................: .
(1 p)
a./
3. Por -la elasL 'cidad-cruzada los bienes pueden ser:
..#thfrhÁW..........i si la relaciÓn dei Precio de
}a demanda de otro prociucto es directa Y b)
sr- la relación del precio de un Producto con la deman a de
otro producto es inversa. (1 P)

4. Marque }a respuesta INCORRECTA (1 p)


sl se trata de rr', bi"r-, de prrmera necesidad, €S de
"l esperar que los vaiores de elasticidad sean reducrdos.
/
XI ros bienes de lu¡ o suelen pl esentar una clemanda
bastante inelástica -

c) Los brenes que t:-enen fácil sustitucrón trenden a


t-ener una demanda más elástica qtre los que no la
t ienen .

d) Los b: enes que tienen una importancra consrderabie en


ei pt.=rp.-,"ito de gasto trenden a tenel una ciernanda
nás elástica que los bienes que cLLentan coÍl una
n: rr j r--i nar- i ón recit;clda .
5. Marque la respuesta CORRECTA: (1 punto)
a) La Restri.ciór,Elpuestaria se ve restringida
únicamente por los precios cie los blenes.
b) uno de los supuestos de la Restricción presupuestaria
indica que la renta se gasta parciarmente en bienee rz
servicios de consumo. '1

c) El cociente de los precios relativos es drferente a ro


largo de una Recta de presupuesto.
a Por Ia Ley de ra utiriáad Marginal Decreciente
, entgndemos que cuando consumimos máJ de un bi-en,
satrsfacción adicional reporta cada unidad demenos ese
bien.
6. Si un consumidor conoce que el_ bien que com ra posee una
elasticidad elástica y le han subido - el precio del
producto, entonces debe esperar que su Gasto
'I'ota_L en el producto. por su parte,
si nsumidor conoce
que el bien que consume posee una ela
0$ quiere disminuir su Gasto Total en Cicho bien,
debe esperar que sólo
dad inelástica y
entonces
precio del producto.
(1 p)
7 Sr Ia e_Zasticidad renta de Ia demanda de1 bien cienominado X
es de -0.65, ¿cuá1 es la interpretación econ ica de este
valor si se estima que la renta de "l-o consumidores
aumentará en 6Z a el siguiente año? (1 p)
:.¡...1*...d*M,

8 Marque dentro del parentesis si c/u de las afirmacrones


siquientes son verdaderas (V) o faj.sas (F): (2 p)
a) trl Efecto-sust,itución es ra varración oue
experimenta er consumo cuando una variacrón del or.a.o
traslada a1 consumidor a una curva de indiferencia más
alta o más baja. (F
b) El Efecto-Renta es ia variación que experimenta ei)
consumo cuando una variación det precio traslada
consumidor a 1o largo de una curva cie indiferenciaar
dada, a un punto en la que tiene una nueva RMS.
( É)
c) La RMS es La relación a la que un
díspuesto a intercambiar un bien oor otro. (\
d) El valor de la Rlf? es er cociente entre er precio)
del bien que se encuentra en e} ele de las ordenadas y
el precio det bien que se encuentra en el eie de las
abscisas. (É)
e) EI Efecto-precjo es igual al Efecto_Tota7. ( v)
!I. DESARROLLE Y/O GRAFT TNTERPRETE DE SER EL Y/o
CONSTGNE SU RESPUESTA ANTE I,AS STGUIENTES SITUACIONES
9. Con l-os dar,os ce cuac::o sigulente encontrar el vaLor de la
elasticidad precr,o de la de¡na¡da (err) der bien X asi coinc ra
elasttcidad cruzada (€c¡r:i.r.:, de la áemanda ciei ]:ien y cuando
varía el preclo Cel bien X. AsrnLrsno, mencione gle ti_p'.q cie
elastictaad precio posee ei bien x y en iunción a ia
elastlcrdad cru:aCa, rndrque que tipo de bienes son lcs
bienes X e Y. Einalmente, grafique Ias curvas de demanda
correspondientes en función a los datos de la tabla: (4 p)
Puntos Precio de]. a Demandada Precio de]- a Demandada
bien X de]- bien X bien Y de bien Y
A 22 20 20 40
B 32 l6 20 30

10. La siguiente tabla muestra cuatro combinaciones de los


bienesXeYpara un determinado consumidor representativo :
Combinación Bienes Cant. Cant. Cant. Cant - Cant. Cant.
X 2 3 4 5 6 1
A
Y 12 6 AE
3.5 3 2.1
X 3 4 5 6 1 B
B
Y t2 B 6.3 5 4.4 4

c
X 5 5.5 6 1 B 9
Y L2 o 1.8 6.1 6 IA

D
X 1 B 9 10 11 t2
Y L2 8.5 1 6.3 5.7 5.3
a) Grafique las curvas de indiferencia de las
combinaciones A, B, C y D en un mismo sistema de ejes
y consigne correctamente los valores. (2 p)
b) Determine la RMS entre todos los puntos consecutivos
de Ias cuatro curvas de indiferencia (Nota: Use tabla
de ayuda abajo mostrada) (2 p)
c) Suponiendo que P,: S/.2, Pv: S/.1 y la Renta (R)
S/. 16 al dia (todo 1o gasta en la adquislción de los
dos bienes); trace en eJ- gráfico de a) Ia Recta de
Presupuesto y consigne los valores en los extremos.
Asimismo, determine y consigne J.as cantidades de
bienes para las cuales el consumidor maximiza su
utilidad o satisfaccíón total (óptimo) (1 p)
d) Si el Px disminuye de S/. 2 a S/. L, permaneciendo
constante eI P, los gustos y la Renta (R) del
consumidor; trace en el gráfico de a) ].a nueva RP y
eonsigne los valores en los extremos. Asimismo,
determine y consigne J.as cantidades para la nueva
situación en la que eI consumidor maximiza su utilidad
o satisfacción total. (1 p)
Nota: T-1-1a de ayuda para desarrollar b)

Cornbinación Combinac. B Combinac. C Combinac. D


A
x Y RMS x Y RMS x Y RMS x Y RMS
2 L2 3 L2 5 L2 7 L2
3 6 4 /B -¿t s.F 9 ,"8 8.5 -5,5
4 4.5 -l5 F 6.3 - l,+ ./e 7.8 - ):* 9 7 -1"6
5 3.5 -A /e 5 -l,b/ 7 6.7 z{, t 10 6.3 ./o,a
6 3 -o,5/ 7 4.4 -ú,b I 5 t -03, 11 s.7 -o,6
7 2.7 -oA I 4 - 0,9 9 s.4 -o,G t2 5.3 -0. t{
Universidad de Piura
Facultad de Ingeniería
tu(
curso: gcoNóMÍa (Eco)
Segunda.Práctica Cal-ificada
Fecha: Viernes 24 de octubre de 2OL4

Ape11idos y Nombres:.
La presente práetiea tiene una duraeión de 1 hora y 45 minuüos'
EI puntaje de cada pregunta se encuentra especificado en eada una
de eIlas.
No puede tener apuntes de clase, fórmulas, ni oüros materiales
sobre el tema. SE ACEPÍA EL USO DE CALCULADORA'

I. RESPONDA, CONSIGNE O RESPUESTA AIiITE


LAS SIGUIENTES PREGUNTAS:

Indique a cuá1es de Las siguientes funciones nos estamos


ref ir:iendo más abajo: (2 P)
a) Función de Coste Varrable Medio '
b) I'unción de Coste Total Medio'
c) Función de Coste Total -

d) Euncrón de Coste Marginal -

e) Recta Isocoste.
f) Ninguna de las anteriores -

A) Relaciona la cantidad de producto con eI mínimo coste .de


proouccion. ( 'g)
B) Drsminuye en tanto es superio:
- -!- -- - - a ry es crecrenre
aI CM.
cuando es inf erior at CMn. ( 'l¿t
C) Muestra el ritmo al que varian los costes de producció4'
(t )

D) Contiene -Las combinaciones de factores que la empresa,


puede adquirir, dado el coste y los precios de loP'
factores (91
E) Su punto minimo que es interceptado con la curva , ce
Costo Marginal se denomina óptimo de expTotación. ( á:l
..:
2. tr1 Benef icio Económico es JC,*..*-V? que el Be,ríef icio Contabfe
porque lcs Costo-c Económicos 3on ...',I.v..V;.*....,...... que los Costos
Contabf es. En e9e contexf,o ,¡' -; Los Costos \co7rf--é¡ntcos se
divrden en ,'o'il| ."r;;rqtfW/. cosf os "\Á*d*?rh1t"""""',
mientras que/ Los Costos 'ContabLes
0.{{ .J.\W.I^W. .rr/ (1 P)

3. Aceptando la Ley de los Rendimientos Decrecientes,


\

Producto
,'¿^--.. F.
,"áI CTECE inicialmente de forma
v pués
Por su parte 1o Costos
Totales crecerán l-nl- c mente a un ritmo
conforme aumenta 1^
Id ntidad producida y después 1o harán a
un ritmo (1 p)
4. Sobre l-a función de Producto Medio y Producto Marginal,
indique sin las expresiones siguientes son verdaderas (v) o
falsas (F). (1 p)
a) En el óptimo técnico, eI Producto Medio es máximo. ( ü)
b) En el máximo técnico, el Producto Medio es menor que
e1 Producto Marqinal. ( t:)
c) En eL máximo técnico, e1 Producto Marginal es minimp..-
*rq d) En el óptimo técnico, se igualan eI Producto Medio/ y ,/
(vY ,
el Producto Marginal. ('t )/
"Estoy perdiendo dinero, tr)ero como he invertido tanto af
camprar una maquinaria, €D este tnomento no puedo permltirme
ef lujo de cerrar mi neqocio" Si escuchamos esta frase de
un empresario que actua en Competencia Perfecta que trata
de maximtzar beneficios, su decisión será: (1 p)
a) Correcta, si está cubriendo sus costos fijos.
b) fncorrecta, porque una empresa que tiene pérdidas
nunca debe de funcionar.
Correcta, Si la empresa cubre sus costes variables y
+, parre de Ios cosLes -ti,os.
Incorrecta, puesto que los gastos que La empresa ha
@ realizado en maquinaria son irrecuperables.

6 En economia la distincicn entre Corto y Largo Pfazo se


establece'. ,{'¿ únicamente.,/atendíendo
:
a la existencia o no de
cVYT¿:Y ,4A.L|L
.."..'.....'r...-r". ^ - rt o
POf SU pá-.u¡ qA rl
JU i r-a que
eruv
' --r '- ^ r^ \-'mla-q de Escala Crecientes si aI varra-r 1a
E ID LE]1 LLU1lI,

cantidad utiltzada de tod.ds ios factores pr:oductivos en una


determinada ....J)r.lr*;\:Y.Lti.1Yj..........., la cap-tadad obtenida de
producLo variu' una .g4m-ü.4ry-.zfl
",1
existirá Economias Ce Esiafa DecrecienLes si aI variar la
cantidad utilizada de todo/1os factores productivos en una
determinada ...r4-f.*W.ffa,pfu..(......., 1a ca;n{idad obtenida de
producto varia J,-, ür-ru ..WW)ttLv..jIñ\t:N- (1 p)

)tttv I resa debe SAIIR de I


merc;rdo, esro s iqn-f ica q.re esrá en e
le oLro -odo, e o eI . W,W,/cuando,st
I

.........r1.1..Cf.11*.........21-......
,....,,,,.,.....)/
ma:<imizador de beneficios- de una e]npresa en Competencia
Perfecta sucede que (1 p)
8' rndique cuál de las siguientes es la afirmación
G.RRECTA:
a) La empresa competitiva nunca produce si tiene (1 P)
nÁ i ,-J
I'vr --.iurua>- ^ .
No empresa competitiva no produce si ros ingresos
\ñ/ :l son inferiores a los costos fi;os.

lo empresa competitlva no produce si 10s costes


fij os son mayores que r-os costes variabfes.
La empresa competitiva no produce si ros ingresos
son inferiores a Ios costes variables.
g. 1"9.S!e cuál de las siguientes
afj-rmaciones es la CORRECTA:
En todos los puntos de la curva de oferta
(1 p)
el mismo beneficio. se obtiene
\N
la La curva de oferta coincide con la curva de costo
Marginal a partir de la producción
máxima
? i, factor productividaá media (o
(o) corresponcliente
pioducto
único medro) de1
variable.
c) La curva de Oferta indica el valor de
productividad marginal del factor trabajo. 1a
d) La variación del precio der trabajo afecta a la
curva de Oferta. "o
gn EL CASO, Y/O
CNSIffi SU NESPUESTA TE tAS .

A 10. una empresa competitiva produce un bien, x/ utrlizando


j i trabajo cor, una tecnorogíá dada por ra función de
, r."', ',' producclón x = 5 + 2L. La nos informa de Que, tanto
\\ \ st produce 9 unidades como"ipr...
si produce 15, obtiene e1 misrno
I oeneficio, aunque no nos dtce si éste es
del trabajo es 6 (w maximo. EI precro
' 6) y ra empresa no trene costos
n";.¡i.rl
fijos.
condicrones
¿Ca_Lcule y determine ^-o-i^
?
^^r
.Hff*..":io:::1"...; J=.:I
(z p)
1L. La función de costes r¡ariables medios de una
es: C\/}fe = (X/5) + 2, donde X = cantid.ad. empresa
Calcular d.e producción.
" , sabiendo eue, para
ducto, e l c_oste total medio (CTMa)
\ '- r!r!
es de 7 unidades monetarias /
(2 p)
L2.nr, empresa de servicios dedicada a la limpieza
^, Yi:
ol lclnas, acrúa como ptecio-acepLanre en e r mér-^^^ de r
precio que oreva-ece us-i i;
r

sus costos totales están dados por ra srguienteo";;;=".


"nffide-
funcrón:
CT = 0.1q'+ L0q + 50. Donde q: cantidad
empresa dec_i de laborar por: semana. cur."ra de hor:as que la
a) v o"tu.*Ine:
¿cuántas horas opiará ra .rpre=u por taborJ-a ra
semana para maximizar beneficios,
,, n,
.J :'

b) Calcule los beneficios máximos de la empresa por


(2 p)
semana.
c) .r=Fi arra astos resultados
(]J-arlvuu y muestre en eI mismo Ia
(2 pl
cur;a de-Of erta de la emPresa '
La presente práctica tiene una duración de t hora y 45 minutos'
nl puntaje de cada pregunta se encuentra especificado en cada' una
de eIIas.
No puede tener apuntes de c1ase, fórmulas, ni otros material-es
sobre e]- tema. SE ACEPTA EL USO DE CATCUI,ADOR'4"
tú r. RESPOI{DA, CONSTGNE O RESPUESTA A$I:rE
r.As STGUTENTES
-EIEG]TNTAS
:

1" La contalciJ.idad nacional- se basa que en la idea de que f_


-Ld

cantidad de actividad econÓmica por medio se realiza en un


período de tiempo puede medirse de tres (3)
métodos, Ios cuales generan medidas idén=t=icai {e
f^
rd
r^..
L r - ^r de
.á"tiO.á ^ ^+"i ...i á¡A o¡anÁmi
actividad ¿:¡ ELlos
económipa' son: E'l-
I I os son: El métOdO de
::thiiq# ::"á;-;;- W-- :-t i)ffinr
-1 F', IT

aer
P)

2. EL PBI nominal- valoriza }a producción a pt":.-1-?3


hace a precios
mientras que eI PBI ReaI lc pBr Por
;';;;;"u,= Deflacror del que mide er
es el índice
"l-ill'l'l

q/ Hit-
_
"i --^-^--r Aa ./ .
¡¡reattos (le
^1 ^

". a*13
*u*f.e:;il}"*d*
i" il;r;'.-;i ry4?Ékrí,."???!'ll"
-. p], indice
..ir^+
consr:midoq *i$?.^tr
TE»ñ es a.l índ'i ce ol-le
rPC, áq sue mid,e eI
niver de preci; ;.*-T;.:1rrñ T-u"*átB" dx§ eaw.n
" tffi;
pPr pNB radica prineipalment'e
3. La diferencia entre a-l
Er L ur \/
J
ol
"- t
bienes y servacaos ,

en que el Primero de e1los, valora los


finales en una
considerando la
e
ffi:ffie@#kffil%ffi;,";:'3:
mientras que eI
pais de los gerrbradores deI producto,considerando la
1o valora
4 de los qeneradores del producto' (1 p)

4" En un pais puede exisLir pLeao qtLeo pero esto no


significa auJencia total de desempleo'leElconoce desempleo
existente. en períod'o¡.'dé n pJ.eno ettr'Leo se como
l";;.ot .*ngrmlkp-...mÑ-.*tt.*...r.".., glle e;s ra suma del Deser4rreo
su parte, e I
¡¡riccional y besqr¡.eo ...gd.ru"Affi,U{nJ*......" " Porentre
DeseqtTeo Cíc7ieo resulta de la diferencia Ia üasa
i\ desqtreo v la tasa de desq,eo
*##e"- { (1 p)

5. El dinero es un coniunto de .:*'tl.Ld..r,:;t9.;I.... de Ia economia que


utilizari los indívicluos normalmente para adquirir bienes y
.ser de tiPo: dinero-
" L11i::i""'^, LL--,*:::
y dinero "
\ú i-'""llil'"r.
6. Mencione las variables con las que
un gobierno puede
emprender acciones de poJ-ítica fiscal.
política monetaria. y acciones de
Fofitica fiscaL: (2 p)

{ Fofi Moneta

ERPRET
tr, **a"*
"o*r-"* sTTUACIONES:
7 ' En una econory. en 1a que Ia propensión Marginal a consumir
(c) es de 0.75, fa tasá impositiva del rmpuesto
es del 208, e1 nivel de la Demanda Autónoma (Á) a ra Renta
E:q>orta'ciones (x) de 35 y ras rmportaciones (M)de 175, las
el Presupuesto PúbLico (pp) era J. 15. Consj_derandode 29. si
Demanda Autónoma, Exportaciones, fmportaciones que la
Presupuesto púbrico están dados en miles y el
Se soficlta calcul_ar y deter*ir. de mj_rlones de s/.
a) er^--+rye-e!g @n 'de
r-ncremento de ra tasa impositiva (t) equiribrio de
iel rmpuesto a
un
Renta de 5 puntos porcentuares. rnterprete 1a
resultado y ruego consigne cuá1 fue la er
el_ incremento de la tasá A. "..r.rr"iá==d"=ál
llegar a 1a producción de_ equilin.i.. ñ;;
b) eue ocurre con la Recaudación de fmpuestos (T) (2 p)
comparar antes y después del aumento de la aI
rmPuesto a la Renta'-@rtado. tasa de1
c) eué ocurrirá con el p (2 p)
aumento de Ia tasa del rmpuesto a r_a Renta. luego del
elresultado..¡: rnterprete
d) nn un soro qráfico- muestre er equilibrio (2 p)
prooffi".i;-i""i.i., de la
aumento de la tasa impositiva. asi como luego del
e) si el gobierno en luqar de hacer ro estabreci_do e p)
decidiera aumentar l_á tasa impositiva (t) del en a),
a Ia nentJEn s puntos porcentuales y además fmpuesto
el Gasto de Gobierno Autónomo (G) en 15. C1,ar1tÉ1ffiaumentar
ahora er iqpacto en la producción de v\au¿¿-
rnterprete eI resultado. "q..rriinrio.
e p)
Producción de Equilibrio: yo (L/ (L_e,(1_t)
XN=X-M = ) ) (Á + )c.I)
PP=T-G-TR Autónomas
APP=AT-AG- ATR Auténomas
UI\IIVEFIS¡DAD DE PIUFIA
T-'
ASIGNATURA I- ({)$owua, 03
[I:t6] NOMBRE
TRABAJo
FECHA
No

06 lul t6
PT: f
ñn t+
FIRMA DEL PROFESOR O AYUDANTE CALIFICACION RECTIFICACION FIRMA DEL PROFESOR
o e=orX

!=o,t
A --[>6
*=35
M, al
(r-t)) +x
I /tr --Z\'y í"' -- C, / t r- c CÁ

\o,-4 .ltt / ,1,,,,==-tzll


\ynt= t-o¡5á45 I A\o'-Tfll;Y
r-65(o,t¡ / \ { I a\o---31ü
V= 4 .ffi\ I \ .fo'.__ A _.lst
t-or6 / \
)o =k'r*'( ) ]d
tt t \o' =
4»*
Yo = fl-¡6-

;
; ;pffi1g.t-*#. tgi*,#ffif
t5i" ^,,^,*;*'oryu ffitM co'/

Hffi
ffp;;*"il ' Lv'¡ vs!\.,r '
2fi".h tr
,$:ffi:.*á-
-J /
{aso^ ' '' tl¡ polür'JürL
"d§- '{^tr
.1'[^'l+ú q^¡' {"R.1lrt ,,\ Jr¿ olq v\all¡ytr¡' oll. tl ]

ü*
- ------
;1 6{ lv) (-'"AL --:---
o.rr¡,^newtq¡ [a tasq
<:=t)Y") ,.MAPciüÑÚLr r awu'rc'rn'[a'
f= {.\o 1, = Lo rm)(9
t:,9 tq) to. ücor^dq.j.tó dq-
f =(-oP)Cq 51r5) l'=loir Ll,¿?5 unnpqq¡tos o\ to3)qaB
#.h"j t¡ ásc.r.r o{r eo r S I
eúlra q- to3,trrSSl
ple- S/. pr ta.a,t [o ,h« ln6toto
'af= l'--r-- to3,t{tqt -qo t5 = (arts ;.^r*.r-h do- H,ntr,
qort -too7'
lo3rt23S - /'t '
I = ltr{,8-tÁ'z'
l-\eo=t\xr6'l
c) t&¡g?t5 hrit*
aPP = §f: 1 ' S,/',:
APP''M
I\ pttsop,^n r\o p'd+[-.» c[ 0.]üff!44^ tci.r( [o. -[o¡c" wn&osü^ñr, ( at vrP hoqra
nfruA{t
{§^tq*"'r^) d," gnlOS tmi d¿ {nr^no.c**rr).t\G\ o. }^.^*-}aa "L,rn e.-!.
{"*,^}1t
'"^S"rC"\ (l* q{*".cbw
}ü ¡afiet5), c( Po:t^'r e. qQÍ
er) drLlr da- J5 va ")) (
V\MII^ ot - i^ lto)ó 'ol¿ S/.'/ l-
I
L- cr.,to."!.
pp) tu,y{\qlt5
t =-*-1'-', u* otq»
pp) Xfitt5 r*rB &
;\pv,cS OLe:OtsS
=
\^^r "^**t*¿"púJ.It
prvrulpuño
' *Q.A¿ '86, I q^1."--
l§-\s6f .

Alraa?s - Y'z'
/ . \\6 )Lq'/.
U 1- [oc¡,F !6,, tt',r.
Y-) c= o¡*b t) = atJ. =o¡fS X fuo'ndo { -- &a'2. o,?r'
- eo''' ñ= l5 \tlo --- li
. t-- =o11

Á --
\?5 \o, - ( ¡rt+ e) l- o¡)só-o¡1S)
<=3? r
,- o¡15(r -ora) '\or= 4 "t3\
fq= f,1 \o= 4 .I8\ { ro¡+s(o¡1})
. 4- orls(ol8) \rio'= 4
PP=t5
\o = ' rs\ ' r-oftY-t8\-/
xN -- )(-N{
yN = 3S-L\ ^
l- oft
-t3\ )o' =4 .l8l
\o, 4
*N=6
ol{
o¡*s /
I .rBl
\o = ?,6' t8 [ Y" =L, t?5H t\t
í1"' t(
\o = \6arb w*\¿s sk- Y"
wu"\orn¿s de:obs.

.*^.hño\^^,
fS sr,rrrtiln^r a..l- cc^,to A,) I tq tu9
t* ,,rr¡tq¡rrnl¡ru, q^^ q.-l- cÑYY qolo t . ¡t"
'ñ. i;ru*ñ -d-r-d;.i,r¿.Gve u,'.l¡rc.':tos lfl^A..r^- a,t¡.r'ins^^tará e
\^ Ti ,{ T :rro.,í, L,q\-,tes 'tuL o"t¡S -qf clso b
il m fa -,rAA"U.ffi "-T:' o,
":'
'P?\ -- j6-f - 1., o>\5
t??. \a,qL?b-
'-'?-r
t6 ^,S-.- ) '
pp J&,od"tr *rü
= .*,'' vvwr'er v
nni\\ovres de:t-1 .^

ñ§ rnni\us o\¿
púhrlco ú*l*^^q'
= fi - ^l
^tt 'la' Q"¡o)§
.ei. flt-^-§""ño
eoi.nn poárn",rx s!6exrro.r
3u- * l:.:.":lli:"t*T"tHXf
*^ruo*. o\*al'¡ pesoncLo
h"JffiI..J;
HY* üffi" c\^Iin^tuto e¡^ \5 p*ñ^.
u,^üqa
gnroñ\^o.h^, ',¡¡ra.-[asq
6^L. Á ¿::YY O*y
5 Ir^A
A
'üiil
6n Ñq¡n, I3)?l
b d-acrr ,
Jr - l'oo'/.
) a-,Qrgs - 7/ '
2( = 8b )tPr'/-
JOo -x. J3ftlt ,

,l\ y
vv

t o¿5
t*i'
6'r,-t
\ Lor¿
(ro*)
\

..1,-,.,.
t{\i}\ qt e/ 5
J_
U]iIVFRSIDAD DE I]UR{
FACULTAD DE INGENIERÍA
ECONOMÍA (ECO)
PRIMER EXAMEN
PIURA, VIERNES 30 DE SETIEMBRE DE
2011 2:00.- 4:00 p.m.
NO]VTBRE

l Explique sobre el tercer principio de la


economía: "Las personas racionales piensan
términos marginales,," (3p) en

2. lJtilizando ei Gráfico adjr_rnto muestre los efectos


de 1a intervención del Estaclo en los siguientes
CASOS;

a. Inposición cle r-rn precio rrláximo. (3p)


b. El EstaCc acrila cor¡,o un oferente n.,ás.
(3p) ,.^cbclr^
,J
Err tocios los casos rluestie .lor'"^."i"r,rles
dei
ccns'iriniclor y rrocLuctor jr,iciajes, posteii,rre-s
y
ios eiectos netos dr- I:s meclic.ias.

Consiciere 1os ,iaio.i <iej cuadio sigrriente:

, P. ' :,-l I::.¡ .i= ir.=ór, ib:rrr., Ca;r:icla.l denarCada ;1.: jarrcn ii,,liü l'l --
(e ir n',re','os soles)
l¡-
!:
110 I ,'
{r
a' si el precio baja c1e sl.'lzá'tu sl.''iio, encuentre Ia
elasticidad precio d-e la
demar-rda de jamón ibérico. (2p)
b' De acuerdo al varor que obtenga cliga
qué tipo de bien es el jamén ibérico" (1p)
c" ¿Le conviene al ernpresario qr,re ei p.ecio aet
¡amon ibérico baje? Explique por
qué. (2p)

4' Considere Ia siguiente función de producción:


X = KLoj, donde X es el bien final y, K y
son los ii-rsumos capitar y mano de L
ob¡a respectivamente. si K es fijo e iguar a
5:

a' Indique qué tipo de rendimientos de producción


tiene el insumo L. Grafique las
funciones de producto Marginal y producto
Medio de L (coloque L en e1 eje
horizontal). (2p)
b' si e1 precio cie ambos insumos es de sl. 2,
¿Cuáles la función de costos del tipo
c: f(x)? (2P)
c' Encuentre
y1l es el costo marginal y costo meciio de X, cuando X es igual a 10.
Indique qué significa en términos económicos
cada valor. (2p)

Total: 20 pr_rntos
r

Ur\IIVEtrISItrIATs trIE f'IL'trIA


,:)
ASIGNATURA r<'c rjl o v!! L.\
,=-- TRAtsAJo No ,á*"r.-ü
lr
NOMBRE _.- 4
-A l\
L\ y_ y),,q2 I ,{i,r\l). Arrarcr-aQ. pp¡i¡1 o '\r) kr\T

trIRMA DEL PROFESOR O AYUDANTE


7o
CALIFICACION RECTIFICACION FIRMA DEL PROFESOR

O ' l¡.: frtrgcL'\c\, r^ctüclccLuc .¡itt*so,*'. yt'lí'r*,i,^as y";rJ,^ crLr" '.


t¡'"'írt''¡""'
:1;;;"1*o ttt
:t:l::fti'*=^ ^
"\t,'""^;o,-
l^ )",^'^^,,
r ¿=¿ ':lL \r, * u{*,,--,^
, "li;.";;-'
- LOwYI [..", fi..S,v?-"r
t li^''':,,:';;; u"-
t- ^-,t^.:r""1*"-'"o l:'^'u
i,-,
,,:ir'^+;
;*, "i",i.t,].-li.,,,
vir-. rp
--t-
,,f,, =,c't''' c'-,.o-'l''b
,-1,,

.,',;,l'l'.,,1n
!!' ,

J[c-'"'r*' ,'-)'i'j:t' 3;¿^))-( ;"1'^n'-1


J ,' -/'i'
;" ' ,-,,',./tc, ,L ,¡,.n.,i ,'r¿ r;, [¡, ' Q""" cL' ¡;i-'t';'o^"1¿4
i-,.*u(-r,,:','I,'.,.',,,,^",|l:.::^i,,^,::<
,.,,! ': ';:::: ,:,: ' "Í'::",f.,1 ]n. ' ,,'_'',,i,"*'\. ,,,,,r,¿ r=

-'
. ,,r ,"1;'-, | ll,".r:
L
1|'un.ii,\ot,l"1ClC,urc,LrC^w¿lc,L:¡,,-¡,,'LCaL'(ijr-,¿)r,-.l - t11t.t t
'i ^trn
5'ío 'l',,,nL> arrlt Cctvil r D ')o u"1ct
I
,iJ , *rLl 'r', 6'.L.'.,, t)'ti ..,o "1 i'''l'"T^^d* t püoc'cJ c,,i tl-ct ,,'t,')''','A' *-,
t)l
' ?.,, L'o't ' .'"''i cL'-c^ ht'(
* u.,"¡""tr-J* *" y .¿.L,,.1r". ,,r'1,, : li^o]""" /u
,J ;)at¿14 Vl.--/^ U,1 D!
{tlü r;Í;
_.--\
Ñi"
c\ru'\l\\n t\.1,,,
e1i,i'""¡''",
'ffio¡c,,rri
. I \ i-
-,,i7rL.,! ,--,rj,, ¡'rlci ).^c-.u
*lú
,^. ñi-¡ rr
/2Ü\re O,..í
c>uct
¡ittlt'"
frc,o ct,
1, u, t,*$p \")
f
o clv¿ú os cL

ea a c^crv-

- t- t ¡
tu
L;)-, s,{ (l L L L, S

5¡;,c.'eclo<d
n,{-\s'
vt e¡ t"lt Leffi ¿l *-i*,,!r !-,-;ie.t-r
la ,y r-^,t- ol C,ur*',. 5 u cl.c,r ",,.,¡
[o

_i.f
pttvtit-^,,ttlo ü1,pWuro uv1íxl*-o
o
I
I
'ul 8,,t ilrrt C-u^-ir rc^r y,te fr uo
,Y\
i"*
I
I ttá .*I o ¿J"tL,r
=^^r 1t.*r a-,1

A
't G,, ñ,o $-z
-i
'( *i, cL, F4r^, lit., .r 3r e i,,,o-r
'4 u, !'¡ , Ü ... '- ü-t f^-t^-,r 1J r.i.¡,.- c,\g.tr .
€ na.,, cL r'Lr.*tl* .
I

* eI C.u^-^.:u,r.;cL,.- Cuw?*(L) ¡(-f,i{¡


'.1
(4 Lo,-
!u*..-.,á CL,1c,*c,lcr,, 1i
r ¡ , /^
-J,,r*\c., -, lr,,,Lr,ii, s, L, 'U' cixcr.ri )C-.'-, C ('(. t,. >c ,-, ras :! 'll )

(=;-- ;\ rcy- ¡ qtel.r, ,L


i¿.i11" .

-> Pr = ? *i*n *\ ew ,b-k ,*f I

.[ L

lf
.,;r,g:!L
vr-ic\D -
// t\ &l co"l,rn^,clc.r = !. t{ Fp, lli
-.-
\-( 1qO
i ) P\ k',-. EC'_- S OF?l/ .J'u'it:-" =
- .J¡

|
ti,,'l : t) A.-
\',, A,k
.c\,. r\ I, clJ{' *r",,r^;c,[a.r' Dñ r{ ,= ,/2
[--./
\
É?'r?, tr d 1rl

¡t|'
cLttlc tU c,lr.d* =po
/'-\
Appr . *tt. "(z
\_/t
¡,c ^

ur) e \ p,i.^.c\¿ ctdíri


Orl§ i'i
YI. A ¡1*Ñr onv¿''e.
et ¿rk^c\p .J C,duct" C-cvrc uq C1¡'"J' wlcí
!)f ici
1r;'o'\r
I ! €'{o,\i
Ctrurt+ ¿ftct. C1.,< {0, Cuv""+^ & o,{z"lo sl
J,e,¡icizc^.; L^c^t:a f c.t cfu a r,lncL
, / f,ocLuct,i
Q" -7-i - -
¡lt,,lV¡(
_-
./ // 1" ln: c^-s.,-"i¿,.; tlr-^,,*.r{.
Q. txJ c'r

¡lt' t ./' .

' uA' Go üt^ r


);r.iw' - ^ )-,-
C^.stcr\o-

f=Ig- C0¡ [¡wL'r-: r' \ovvj0r.,*ci., .Lr] Coqs.,,-rc,fu-. = po A E


ft, =0 €)O
(U) ¡.tcL"d0¡' roAtP¡' -O
e? = ?. of, ¿p=ft i r/'J¡í"["tu
)-Cn,i . O O > -lO*6 > G:Jo x¡r**lo Qr Loj ,,"¡, o¡c*c^c-cu ,¡x^,t aI
C¡§^.t:u -.cL;,r- ,
?r- lp I-a.. {, ,)f" L\)¡ c\c{a L> Lri*
a
a ,i fu;u,"^tou,
€ t **
¡" I Ltrc^^ ,vt d, e s+",ñ 1^y. | ,L' tf ! v§
fe,
I\ O. }t , 7ZC{)
*;-ltc , I rtc+ \
- \sot )[ :,w[ ii;;
>\ n -\ZzU)
'l lY= \
) ri
-)t lt) =
-, ix. *Y' L
/¡',)I
¿--l \
\z- {(it tro -l2D) [ tw) +zzrD\ - (
', Jtsoi
\-z_)
u\I t/^-
tr
l.' -
« fr¿.r'¡-¡
:-tv)\>¿---

\nr^-.
_ -Li ,55t35 -> Itl tI,
, ,[ ,/ !/,:rS
tt v
l35.
* jh\iT )
t r)y

, G \4,1', uic\t',I P'r


c't tLr !-c'

¿--trtí .1 L¿tl¿r- c'J:sol"''1-''


i Ftá'\'':
U) rl . i-o u '") L)t J*"t'
ll
[=.-i,]5rji 7\ v1¡':,'lorr/'il-" cL+1 D*""u('r '
,), \ J,/ 'l
/
a\
\
/) a ; b 1*'t t' ' z, ;
r n'
rl'.-lirl
C ) l-u,'- 1'. ',i i a,'i i'' Lr r-;1 r--;, iv' ofás'hcÉ ) Y\
Ñ Luaa 'Ll -!- r',,r )Cr '-

,;li u'ci ,r{ u'1 i'1{:r;e''


c
Ú:-'i,
i \l ,- c\'t'
,l,,,1 \r\ro'". ',1¡;.c r,,.-"
ie fa'?u
!, fr<(", J-':'* r J i .1, [c" vv\ s.,'[, u'
JL'-u
<"-

'r-.,i'¿''-, l" ':fO podT.'nt,C5 t*"'",f'av>c^^r -)l


\
-l -' ,-'',

*/ tr\i t-r D *-

A íh,pu-- --
-[r ?,, , X »
I rl ¡
! [.,rvrC-<'!¡ r¡
,'f.¡ *" LJ'f
L"¡^ +i l"
- lln oJ, tc\,c) L)(t9 ClC"1t': t c,Lt
4Wv\/v/uv I
' C¡^,.v¡r--t C t[*c'' -'rS L, fJo
fo'
Zl\l-Ql).o I ctLlc*'¿ LN-1

5¡ ',
nc¡rtr:'o c*|^nr'q','{cl '

: .:'
.'f\;, .
i
w. .l
\\C Xt , Z¿CD

U \- I \ r !
I .-^l fo
¿:rtí. ¡ , ^ i-
,\,1 i'Ctl-r''r- tü'sulw\- '

,\\ :\z ./Ylrr"lLv


i_
,") L,l f =q,35
il
Ij: >l Po. =(
I

,1 {rc¡tle-
r: a\
1
+'F x ?x
t:-'l €"' b L}+r'v;t'y
J''r^a¡¡'t{'tcx*r-;
rf'Jl:r l - ^Jr-..
t ,t-- )
r,v1 ,1, Lc*,o
'
c i li,t', - 'Y1"i1¡'' tL' u'i !2'i'
,, '-\
P.{GS
' ^nnu u &:Í $J t*ci ,r{ u'n tY *)*'
\ t"', J'?"* ',\2Lü.r tr.¡e P"'\
!'t ^-' f\(ui
'-'Jc ru p"ía"u-i,r"i'lr-"-,rl;^-
oLr Lc^' ''¿'**u- + "

\,i rti.'; "')

rx " K +
J

\?c.; x Ii{1} - t.jCQ--a't)


, inr*l o *+
/-\

^ lb,,n""- -4 ?o , X +
/-\
IIO * ZzCc =Z'1 zaQD
l' '
I
tl

r
- [,^-r..¡,^-<b ,' I

,f-i
"" Lr"¡o
-l a
1.,, , lr'\ lJ
c'LL
l" 1-u\'+r^u
lU I

i,rq' Y(}.o' , [' ro'


OCa 't) 6l' ¡Cr r r"t
f\- ¡
\J\
?ttzr" +"- CL'
r ,
cl
-) , ', nc'rr?-bÚ ttf¡*nr'q"tl "

rJ

\j- r'.1 '


F n(
r \ ut'
*) l-I
t--/
I

il
lY,, - \.' -rl
i*)
?n*. - )
¡! L
L
I
I I r'/-
L- L-
L¿, t))< L//
,/
j/ \,f",
ilI\t.- - L, ), L3e
a .- r.- /l
'' L'''t
I,t\L :/_\\,. ,iv : lrr, )l/
=
L¡t ¿t n
YfteL
' ,l
r)
.. ,r) Ii¡tL
-7 /l\¿u ) tir.,i L 5
l*y \,f,
3 !.r.i.r cL ¿ "r . -t
rLd\tLrvl\rdvri¿,5 i.
cU
1(
Lt )

¡.uc[,_,.rctú" JlrCl.f L\f ¡, l¡'" ?pt,L


L
r\-
url L - \''.. \( , ?o = ?u:z
\q_.:
/'i..
Z Ia) +
.,L
L= tC +

U
¡C\rC,. X = \O,
I

1C_ ir¡
e)
-\ \J + -,¿\!^,/
*1 -,\
'>/ l-tl¿.
(C zs
=

4§ -
vg .() f,-
, pCa rc' \
¿\Á ./x I

:7 q (iD)
l,
CJ
=

[,( + *'? r ('L'iü tlr^ !*'


t/

/ Cv\n, 5;c\
,Jt rar l-J rc,\ ri to:\o
f
\)^i c'lack' ( ULt a.'({ cV

zc,t13
= \,b ) á3*^1¡.^ ,i ks.|o fr)r t o.c,la u,.'^ c('c.J t[t '
ci ¿1c'- -
f'o
UNIVEFTs;IE'AtrI trIE PII.IFlA

ASTGNATURA €l)rror-r.A_ TRABAJO NO /,|))-


N9MBRE karlcL ó,rrrios A,',',:''¿'rt FECHA 13 f c"¡ i':cit|

,lIr
i
Y
FIRMA DEL PROFESOFI O AYUDANTE CALIFICACION R ECTIFICACION FIRMA DEL PROFESOR

(Ü cC, c -Zoo 7 -- - I O C<:a rLc,-r'f> ,,


- tAcO:) tlaJ) e /
\
aCcÓó az,ó?? ,'''
P';3}O'dilc'x'r t //
CíI) Qd= z<,.t:c-
,¡t/ fj ,i
{io: 2t1o:)
) ¡,rt

Qd.-o -* P= 4oc
f , O --¡' Q¿f ": f,,r co:,
Q.r':'rr -P f): 4OC

f;¿:--i G,:s='iCaLL
'1. /

t.c-- !4¿''s'a,i3)iz)C'C'I1 'r'.../., -,r ,


t --,t " / \' -
a,
c'l

É P= \ r": -,?- ) :32_?, _lr, r r,.1, --, -


-
g7
LT: (t'+ ¿ p- S'o<;.2 OCo

L,{: - A(v N " .Jt-¡


- /'/f/'
-
-{in o
á - ) +' eÍath
#rr=
¿" - -4 ^'u
q... -- L(,d x
19A =3 -. \soy';.,
.^t ?Wp 1= ''-
Ap,= 15
al t 5'?z) ^6c.zc
I'' /I (\/
\\
k \ 'P. \oc t ap: ]{rc*
--

flg = ?0o - (oo-^f l \-.---J

(') --H;Ia \,() ,/


?c. &("'g xr<>lr*- 5rs'L
i,ufr,u-ar*, y't c-rt*u¿ pl ¡'odwlua''
\/"rzrzr¿r1{rw
l'v - .y -r,,o-L . 6L T ?| I ,':u c'-,'t^"*lA
Vo Tn^ /, ,).J,r)n¿-,,t- .c!t /.,, ..V.**- j.^.,1^.^e-M-<,r
/
/i'.)
l\ 1/
1/
tl
V
'r
\{

I¡'rgtzrol lt\'a,\c,
'bO x Lb ,'oo
In
#
W¿h,e*, L'q".1'J, cl)<) [."k'
\
I
-n. -/ rf r3¡.J,ú*üs'*
- t,n¡ 'f' ¡,n*y

t,L h*h+ r, A
*( f'n''i-)
1rl A-
.t r ¡,\ r I
¡ l*\ (,p
ilL) )

q ^!-, )
I

lr
{\,
".,:i
¿t?,)' ¿rxaé.

-*l
at ArB.a7{o*i it*C-r¡
p '/
i¿+D.€+r=,( {rD)
k d*u 'Lorru',

iz I
Z.
Iri \ \
[" /
tl
I \
i. \
Cl
(¿

'i r'. r-
1 \

"\ \.,) Va!üY,,^/ U to;*r.k^,,-,.,, t<pf"


_t, \\\\
? 4L =i,x
x (,.1

l2
*l[
!q,r) -_,1.J

4
{.y*"e** .4 ¡¿ 7,'
t-r, lt, : l(,r.
5
. --7 - ;/l>.
lt
u-v iL, t/
'.
\J .ll
--:LU''
r'\r
, .-\
/ I \'\-
Universidctd de Piura
h ,'!"Y
r xrC \_-/
Faculteul de I n geniería
1u'\
\rr
c'uRSo: nco¡¡ouÍe- Eco
ssccróN \\A"
EXAMEN PARCIAI,
pruRA, tutÉncolus, 02,DE ocruBRE DE 2013; 08 : 00 AI'f .
ApELLrDosy NoMBRES:. . /,v$c.s" "r4.tras.u-¿-.. Yaa-ta-.' .

La presente práctica tiene una duración de 3 horas-


E1 puntaje d.e cada pregunta se encuentra especifícado en cada una
de el1as.
No puede tener apuntes de clase, fórmuIas, ni otros materiales
sobre el tema . SE ACEPTA EL USO DE CAICULADORiA "

I. RESPO}TDA, CONSIGI{E O },'¡TRQT'E DE SER EL CASO SU RESPUESTA ANTE


I*AS SIGUIENTES PREGUNTAS :

Indrque cómo se desplazará la curva de cier.anca del rnercadc


cle un bien normaJ(derecha-D, izquj-erda*T, no se desplaza-
.
L n:r-]n- l n¡-] \ <; ( I pupt¡ )
L\l')t ^-l^----l
..j-l.iLlCLCLltt lC.uU -tt-.) J-"
a) Ei p::ecio de l os bienes susl- itutos bai a . (F) -1
b) La población aurnenta. (¡D)L
c) L,ls gustos ciel pírbl ico se dirigen hacia c;tros i -ones. ..'
A*i\ b.,

( r )t'
['\r d) EL precio de los bienes complementarjos ba1a. (JJ \I
\l\
e) Se clist-ribución de la renta se hace más equi :1 i.r¡:
iLUL+/vgo
(ryo
(D
,t-L'
) /
f) La renta nacional aur¡enta. ) ",
g) Los bienes complementarios se enclarecen. ( I )i,' \i,1 ....,"
h) Los bienes sustitutos se encarecen- ( D \.,,'
2. Considere eI caso de un consumídor que adquiere dos Jt ^*^fl5
biene
-_ cllt=
(x e y) , los cuales presentan curvas Ce demanCa
dec,recientes. Indique, en cada una de las situar:j,ones que
se plantean más aba;o, de qué tipos de bienes se t-rata'
(1 punto)
a) Bien inferior.
b) Bien de primera necesrdad.
c) Bien de Lujo.
.A d) Bienes independientes.
e) Bienes complementarios .

_*\x. f) Bienes sustitutos.


\\ s) Ninguno de los anteriores.
Un aumento en Ia renta del consumidor desplaza la curva de
demanda del bien x hacia la izquierda. (c)
Un aumento del precio det bien xt deja inalterada la
cantidad demandada del bien Y. (d')t/
Ante un aumento de renta, la cantidad demandada del bien v
aumenta en una proporción mayor que la renta - (v
l
l
Un a¡-rmenlc en ,ol preci o clei br e:¡ ;,ir Ouo\¡CCa un
clespla zami enro hacra ia derecha cie la c.,rt:va de Cer,anda del I
bien y. tft
T,a cLlrr,,a cle Cenranda cr:uzacia ce ar¡.itos bienes presenta T
U.na
pendiente negatrva.
at9-)
Í
3. Se puecle asr qnar
vaior únicc cle eiasr.ci dad para tolu
un
u de denanda ¿Verdaclero o farso? (0.5 puntos)
tC
,- t,.l V\ De otro lado, en una curva cie demanda iineal y UUU:=UAUl1LC/
A -'---^^] ^^+ ^
(l\.2 la eIasticida/ precio de la dem.anda es disr i nta en cada
i:unto Ce la n7i sma . ¿.Verclacjer o o falso? (0.5 puntos)
...Ver.$.u,cie..{ ), /.... .

l/
I

4. un análisis económico reciente, efectuacio en un país


europeo / recoge ias estimaciones real i zadas sobre ei r,,a10r
de algulas el-asticidacres correspondientes a cr¡,.ersos blenes
y servr clos . En concreto, se conocen lcs siEui en--es datos :
Eiast icidac-pr ecic de los servicios cle taxi - L " zi) y
Eiasticidad-preclo de ra erectricidaci : - 0.13. Resoonda lo
sigi-iiente: (1 punto)
a) ¿cuál es el signif i cado econónico cle es-*os valoi:es ?
¿Qué argumenros se pueden empLear para ju-siifica::tos?
b) ¿Por- qr-ié se d-ict-, gue a -ros consL;midores 1es interesa
q.re lcs producios que adqui-ei:en en el r,e::cadc t--ngan
valo;:es altos de ef asticlcad-precro y, sin e'i:arool a
\ tt:..f :?o-i,:t"?j:? les
_ JJ _vLUrrf,vLUlC5 L:5 : r.:e:eSa que
O*i tensan ,,ra_ores
r-:-lgan .¡a ores bar j os ?
b:r;os? -r
^:-n:e-í9ia
.9..). :.ys..#.1..4..(.tss....c.4i....1p.:.(i. ',. :.+?9.. *..*lgi.fr.r.*.'...... .. .".
f l-.1 .l ¡\ 1
, ... F..1.:. t. :fr.li*s.:L...,... .:.. a_.. t 3....* *:1?./*y., h c*..,
W

5- Ivlarque 1a respuesta correcta. un consumidor se encuentra en


una situación de equifibrio y pasa a otra situación de
\*"t satrsfacción. áeué ha podido producir taL cambio en
Fr:rtuación de equitibrio. (1 punto)
Un
,/ U precros. aumento en la renta manteniéndose cc¡nstantes ros
\q b)
,un
oaoa.
movimiento a lo largo de ra recta de presupuesto
c) un aumento de ros precios en la misma proporción,
manteniéndose constante Ia renta "
d) Todas las anteriores _

e) Ninguna de las anteriores


6. en de eIIT:'nadO
_L

rra puede no
la ci.ei,echa, e .r

hcC-o ia i:q'^-.'rd-. (1 punto)


1t:::l!.y.. ..{.w.Jtrt¿.:ut ¿
\r \/ ,d-u.¡.tt",*rA ..

'v\
..]4. /t4*Ala.....
#+i- "4*..

7. Marque Ia alternativa correcta. consiclere e1 caso de un


ccnsr,*iaor que desr-ina toda su renta aL consumo de
productos, a y c. Dicho consumiior adquier:e ac1_ualmente una
cantidad tal de ios dos bienes que se curnple que la
utilidad marginal que le reporta 1a úil-ima uniciad consumi-cia
deJ'
_ -/
bien a es el- doble de ,la Liti lrdad marginal del bien c-
Ey cons'-rnidor estará en equi lrbrtc .si: (L punto)
/^t :- pre.ro Ce a esrr C=I ¡recio de c.
ljl precio de a es el ciobie del precio cie c.
V. \/- O
c)
d)
ir,nbos tienen el rnismo precio"
EJ- precio de c es el doble del pr:ecio cie a.
e) lJ,inquna de las anteri ores.
B"Scbre ia f uncrón de f:rcCucto medio y r:roCucto m.arg:na 1 ,
indiquc s j:t l;s expl:e"i ones slculentes son verdaCei:as (V) o
falsas (F). (1 punto)
a) En el óptii;to técnic'o, eI pr:oducto med"i o es máximo" (V ) ,/
..ahq b) En el- má>:imo técnico, el producto medio es mayol: qlte
1\ \ r \ ei product-c marginal. ( V ) "r//
\ c) En el ópttmo t-Ácnico, se igualan el producto medio y
el producto marginal. (U )/
d) En el máximo técnico, el producto marginal es minim¡l
(Y-tt
9. rndique a cuáles de las siguientes funciones nos
refiriendc más abajo: (1"=tánlo,
punto)
a) Recta i socoste.
, .fV\ b) Función de coste marginal.
\r. c)
d)
Función de coste total medio.
Función de cos:e to[al.
e) Función de coste variable medio
f) Ninguna de las anteriores.
Relaciona la cantidad de pr:oducto con el minimo coste dÉ
produccjón. (\}'i
Disminuye en tanto es superior al cMn y es creciente cuaádo
es inferior al CMn. (e )
Muestra eI ritmo aI que varian los costes de producció4.
C}O
contiene las conüinaci-ones de factores que la empresa pu6de
adquirir, dado el coste y los precios de 1os factores. ('üQ
Su punto minimo se denomina óptimo Ce expTotación. ( +P
3\
¿4

/.
L0". Una empresa en Compete r-crl . tdrf ecta debe CER.RAR.
TtsMpoRALt4ENTE (Cr ¡ sI L ,e/r,: .' y debe sAlril

rN I
,k
/t\
del rnercactc (LP ) cr-rando.... .Y....X4-.f.
^(

de producción maximizador ce Oene¿trcios * fie


El nivef
empl:esa en

\)l Competencia Perf ecta sucede cuando'.........'.J:nf'#- -o-''"""""""" " " "'.'
(1 punt,o)

CoNSTENS SU RESPUESTA ANTE LAS SIGUIENTES SITUACTONES:

l-L. L,os gustos de un consumrdor Se Conocen por la f LlnciÓn


de utilrdac srgurente: U = t2;¡ + 14) (5Y + Ls¡].tz
siendo su Renta Monetaria (R) cle 224 unicades monetarias,
ei precio del bien x es cle 4 unidades monetari as y ei
precio del bien y, de 6 unidades r¡,onetai:ias" Al respecLo
calcuLe v determlne:
a) La situaci ón de equ riibrio de este consumidor:.
\ (1- punto)
t,6
lr
b) si eI preci c del b:-en y se niodif ca
i pasando
uni dades ncneta::ias , LCá1 cule 1z determine la nueva
a ser 8
si'.uación cle equilibi:io? (1 punto)
i\r), 1 re'rLa (trR) y'
'ti il
i c) El efecto sustitacion (ES), efecto
efectc tota-¿ (ET) " (1- Punto)
d) En funciÓn al efectc total (trT) v del efecto renia
(Ei1) , realice ei anáLisis respecto ce los bi enes x e y
tai como se reaiizó en clase. (1 punt'o)
e) E" g"-_!"]"-gIéricc muestre las situa"crones a), b) -v
c) ' (1- Punto)

L2. La función oe costes variabLes medios cre una ernpresa


es: C\&fe = (X.ls) + 2, donde X = cantidad de producción"
Ca-Lcular el óptimo @, sabienCo Que , palra
ffi@, el coste t.otal meclic (crMe)
es de 7 unidades monetarias (2 puntos)

13. Una empresa de servicios dedicada a Ia limpieza de


of cinas, uttú. como precio-aceptante en el mercado . trl
.i

precio que prevalece en éf mercado es de US $ 30 por hora -


Sus costos totales están dados por la siguiente funciÓn:
CT = 0.1q2* toq + 50. Donde q : cantidad de horas que la
empresa decide laborar por semana. Ca1cule y determine:
a) ¿cuántas horas optará la empresa por laborar a la
pa., *u"imlzar ganancias? (1 punto)
b) "e*.r,.
calcule las ganancias máximas de la empresa por
semana' (1 Punto)
c) Grafique estos resultados Y muestre en el mismo Ia
curva de oferta de La empresa - (1 puato)
l.:»i t,er¡ id¿¿t/ ¿/e Pi¿¿t«
f'uc tt ltod de I ngetr iarío

Curso: ECCNÓMIA (ECo)


Segunda Práctica Calificada
Fecha: Viernes 13 de setiembre de 2 0r-3

La presente práctica tiene una d.uración d.e L hora y 40 minutos.


EJ. puntaje de cada pregunta se encuentra especificado
en ca.ia una
de e1Ia.s.
No puede tener apuntes de cJ.ase, fórmuJ-as, ñi otros material.es
sobre e1 tema. SE ACEPTA E]. USO DE CAICUI.ADORA.

I. RESPONDA, CONSIGA{E O DE SER EL CASO SU RESPUESTA ANIrE


LAS STGUTENTES PREGUNTAS :

l-. Cuánios más bienes sustitutos ex_i an de un bienr ieué le


ocurri-rá a la elasticiclad -pr..rq/d, la dem.ancla?
1acio, ¿pór qué S-a dice que u 7os consumicicr:es les ir-iteresa De otro
productcs_ que en e1 mercaclo tengar-r
lff
,-\
:::^
!'aloresli= alcos
-.--^^,,-^r
pr-ocuc+-ore*s
.aCquier:en
cle elasticid¡r.l
les in-r-er:esa Cr"
-nro¡in r/
-
gur\'
iery{an
,/
.rin enbargo,
!t., sin --;.;;-^ a los
*-)"*
"=1;;;;-'1,.i¡.q,
(2 puntos)
-

"<\-¡tP*

2" Pcr st¿c¿ daC-renta los pueden ser: a)


........... e b) er
^l frr 11[u!
^-i-.--
CASO, si, el valor de la elastici
t,wIftP entre 0y t, s seran:
d-renta encuentra
............... y si e1
va l rtr que i, entonces bienes serán
(L punto)
3. Po asticidaC*cruzada l-os bienes pueden ser: a)
f
si la refación del precio de un prod
.;/;#í;;; ;;' otro producto es directa y b) con

0\ si la relación del precio de un producto con 1a e


otro producto es inversa. de
punto)
4.Ygggg .l-a respuesta INCORRECTA: (1 punto)
@ un subsidio otáig"d" p". er gobierno incentiva ar
cpr?ductor y desplaza ra curva de oferta de dicho bien
-4¿ a Ia derecha.
\q ffi de
/*
si la relación: Elasticidad cie la demancia/ Elasricidad
Ia oferta, eS pequeñai entonces el subsidio o
subvención beneficia mayormente a los productores.
c) La proporción de un subsidio que terminan recibiendo
los consumidores o productores, dependerá de ras
.t^ \

Y rr demano¿'
1a oierta ':-----=rte
IC Y -
deI-
j ecae :"i:^.t",L J"..Jil- *=,."u
) que tenqa

e iá st ica (1 Punto)
rei na' ''^"':'''::!::
5 " Nra-vq':e j. ':'o;:;" r:T=. 'r'-¡u-' niuluo' - ie ese
, ,e -:y:"::,i;;:;;:
'"-.."^;.*cs o-ue &cl}( *v''-* ''liJ'."
=;:.tsraa.lo:";,::'';,"
\... V -ó , ve Leslr:-ngrd'
!\\
'Y' ;1:-il'.'' =:,*:..?j'io = 1","J;'"*:'.1:1,
u) :-a Rest::-;;r' ros pr.,"Y. ;;=!:^::lL"n." err bienes v a1
e) :::'XT'i:i
vt q" ll;;::::=,1" n.l.-u
Pa::r
a l¡
i:rdtaa --r-¡-q ?s dl:eren--e
servici-os cle
.:"1::":ru"io, relatrvcs
u' ir .".:"^":;r"?".",-u
¿) '* Pres':puesLo"
de una ñcu u*
tutgo 1/ T,areo Pla zo :.:

"' \1in/cutL''""- Por >u v*- al var


"=au5la.e r,..""'.:.i
o.arto"r r,f: k "#tiffir,', S: .::;:, i . ; :.';.+::, S;:
§Q i;;;;;';,".?^,..J-'.,..,,,,,ffi,I?-:".=l-i]Í..,*.,
o¿
:^i?:::: :í:l*;i [."#-iJ== ';"o"',:;;o" '.: ?';':.
,.1.- ,]?^J)"'ffi*)
(á puntos)
l.;it;; i'^tt p s.tpn¿,.i-
[H-r';"
::?:;,;;;var ,:ia ?;en una ,)?^""2,"tffi*; (2 P""to=)

;:;;;;

..i:#ür:".+,*;¡
' ii::;"-=,;,. ;:tf;"r"f.",#,li?i;lj="1 o La ca:r

r.t'"?'; d" e n ":,';11"" =. r.'ri..,i'.",


.l ¡jii-=.;1 v#*r.*.,1^:l-
per?lTllT;iá;ffiado,
o..:ÍT:??- " -á;i
"'." "'j",
:,produc-uoi,
ll .r:r*""
sv> produc-uol-,
lf :11
(2 ^,,.,ros)
n""""=l
_equrrt?t.i:- del Excedenre l:",:""-,
/óo.r=r*idor, , ^r imouesto de US '

ü**T*t+*á:*='F;H¿
3: l: :''' :" " (1
;
uánr o
.' 1,"*-*, .' xi?: . "*
u) i::i"h?,.'="J.i""?'xáol"te
¿c
J :Üdel xxff
Proout-'"'-
¿
i lt'11i::
Punto)
v eI oer iiscate-'
scaru? ' ..- r
nore sos ffiscales?
^:- iingresos
Iio=
lo!-t"nresos ^r del nuevo P..11t*?J"::i
l1t=".".,¡, cleI
:^
Oe
.;;;;:
Á" .r";
",,-ii,' l:
!s
;
: : i
-:
::
soclcqra*'
" ñ ól
^nrre
t::::1o",
er
" ":l=
ill ^".X:l::
" .:"
COSte GC
;
: :' :;:l:l:?..1"
.? i:'-r:i:? . i' ,-SXlii:.
uus-:.=ffi *
hi enes ua' " de a )
La sigLriente tal-¡ia nuestr"a i:uatro com.binaciones ce ios
bienesXeYparaun cieter minaiio consumlCor represencativo :

CombrrLación D- ^*^^
UICJlCJ Cant Cant Cant Cant t-:1 1 L Car: r
x L 3 4 5 6
A
Y 72 6 4.5 3.5 3 2.-t
x J 4 5 c 1 B
B
Y !2 8 6.3 5 4.4 4

C
X 5 5.5 6 1 B
C

Y l2 9 1.8 6.1 6 5.4


a
D
x B 9 10 11 L2
Y L2 8.5 1 6.3 5.1 5.3
a) Grafigue las curvas de indiferencia de las
combinaciones A, B, C y D en un mismo sistem,a de e;es
y consigne correctamente los valores. (2 puntos)
b) Determine la RMS entre todcs los puntcs consecutivos
Ce ias cuatro curvas de rndrferencia (Use tabla de
,.,,,-t - drrd
dyutrd .l----- '_^-, --^-\ )
JU LllU>LLa'J-7 . (1 punto)
c) Suponiendo que P" : S/. 2, Py : s/. 1 y la Renia (R)
S/ . L6 al día (tocic I o gasta tcdc en la aCqu-rsición de
ios cics bienes); trace en el gráfico de a) 1a Recta de
Presupuesto y consigne ios valores en los extrenos "

isir,..isno, determine y consig'ne Ias cantidaCes de


li.r,". p.ru l as cua Les el consumidc;: r.axinr.i za sL1
iitilidai o satrsfacción to-.al- (óptimo). (1 punto)
ci) Si el Px clisninul;'e j-, 3/" 2 a S/" 1, pe-rnaneciendo
c,f rstanr: e e l P, i os cr-l.rstos _"" .l a Rerrta (R) del
consunídoi:; trace en e1 gráfico de a) Ia nueva RP y
consigne los valores en ios extrenos. Asim.i s;no,
deLermine y consigne las cantidades para .l a nueva
si-.-uaci:n en Ia c-le ei co:sun--cior naxini-za su utiliCaC
o sat: sfac:ión roIai (1 punto)
Se sugiere que para desarroJ.J.ar b) utiJ.ice 1a tab1a de
a1.uda que se encuentra tríneas abajo:

Combinación Combinac. B Combinac. C Combinac. D


A
x Y RMS x Y ¡úrs x Y. RMS x Y-- RMS
2 72 3 L7 5 z/tz 7 ,t 7-2
3 6 (ü 4 /8 !- s.l 9 ,: (u ,"A 8.5 \3J !?
4 4.5 \i [.s 5 6.3 rA ./6 7.8 7.+ 9 7 lJ.5
5 3.5 1_ \/6 5 r.j 7 6.7 t.,l 10 6.3 b.t
6 3 >"s 7 4.4 t'b 8 6 ).1 11 5.7 -lo. c
7 2.7 ),b I 4 o-4 9 5.4 o.ü L2 5.3 / b.+

S-ar putea să vă placă și